introduccion a las ecuaciones...

136
TEMA N o 1 INTRODUCCION A LAS ECUACIONES DIFERENCIALES Denicin. Una ecuacin diferencial, es una ecuacin que establece una relacin de una o mÆs varibales dependientes y sus derivadas con respecto a una o mÆs variables independientes. Clasicacin de las Ecuaciones Diferenciales. Las ecuaciones diferenciales se clasican de acuerdo, con el tipo, el orden y la linealidad. Clasicacin Segœn el Tipo. a). Ecuaciones diferenciales ordinarias (E.D.O.). Una ecuacin diferencial ordinaria, es una ecuacin que solo contiene derivadas ordinarias de una o mÆs variables dependientes con respecto a una sola variable independiente. Una ecuacin diferencial ordinaria, de variable independiente x y una sola varibale dependiente y, simbolicamente podemos escribir F x; y; dy dx ; d 2 y dx 2 ; :::; d n y dx n =0 bien F x; y; y 0 ;y 00 ;y 000 ; :::; y (n) =0, donde F es una funcin a especicar. Ejemplos. a). dy dx =3x +1 E.D.O. de varibale independiente x y dependiente y. b). y 00 + xy 0 =3 E.D.O. de variable independiente x y dependiente y. c). d 2 y dt 2 at =0 E.D.O. de variable independiente t y dependiente y. 1

Upload: others

Post on 17-Oct-2019

7 views

Category:

Documents


0 download

TRANSCRIPT

TEMA No 1

INTRODUCCION A LAS ECUACIONESDIFERENCIALES

De�nición.Una ecuación diferencial, es una ecuación que establece una relación de una

o más varibales dependientes y sus derivadas con respecto a una o más variablesindependientes.

Clasi�cación de las Ecuaciones Diferenciales.Las ecuaciones diferenciales se clasi�can de acuerdo, con el tipo, el orden y

la linealidad.

Clasi�cación Según el Tipo.

a). Ecuaciones diferenciales ordinarias (E.D.O.).Una ecuación diferencial ordinaria, es una ecuación que solo contiene derivadas

ordinarias de una o más variables dependientes con respecto a una sola variableindependiente.

Una ecuación diferencial ordinaria, de variable independiente x y una solavaribale dependiente y, simbolicamente podemos escribir

F

�x; y;

dy

dx;d2y

dx2; :::;

dny

dxn

�= 0

ó bien

F�x; y; y0; y00; y000; :::; y(n)

�= 0,

donde F es una función a especi�car.

Ejemplos.

a).dy

dx= 3x+ 1 E.D.O. de varibale independiente x y dependiente y.

b). y00 + xy0 = 3 E.D.O. de variable independiente x y dependiente y.

c).d2y

dt2� at = 0 E.D.O. de variable independiente t y dependiente y.

1

d). y{v � y0 = C E.D.O. de variable independiente x y dependiente y,donde C es una constante.

e).�x2 � y2

�dx� xydy = 0 ,

xydy =�x2 � y2

�dx ,

dy

dx=x2 � y2xy

.

E.D.O. de variable independiente x y dependiente y.b). Ecuaciones Diferenciales Parciales (E.D.P.).Una ecuación diferencial parcial, es una ecuación que contiene derivadas

parciales, de una o más variables dependientes respecto de dos o más variablesindependientes.

Ejemplos.

a).@u

@x� @u@y

= 0 .

E.D.P. , variables independientes x, y; variable dependiente u.

b). x@u

@x+ y

@v

@y= C

E.D.P. , variables independientes x, y; variable dependiente u, v y C es unaCtte.

c).@2u

@t2+ t2

@2v

@w2= t

E.D.P. , variables independientes t, w; variable dependiente u, v.

d).@2w

@x2+@2w

@y2+@2w

@z2= 0 .

E.D.P. , variables independientes x, y ,y z; variable dependiente w. Es laecuación de Laplace.

Clasi�cación Según el Orden.El orden de una ecuación diferencial, es el orden de la derivada más alta,

que aparece o que se encuentra en la ecuación.Una ecuación diferencial ordinaria de orden n, de variable independiente x

y variable dependiente y, se escribe:

2

F�x; y; y0; y00; y000; :::; y(n)

�= 0.

Ejemplos.

a). xdy + ydy = 0 . E.D.O. de primer orden.

b).d2y

dx2+ x

dy

dx+ x2y = 0 . E.D.O. de segundo orden.

c). (y0)2 + xy00 + y = 0 . E.D.O. de segundo orden.

d).@2u

@t2� @2y

@w2= t . E.D.P. de segundo orden.

Clasi�cación Según la Linealidad y No Linealidad.

a). Ecuación Diferencial Lineal.

Una E.D.O. lineal, de variable dependiente y y variable independiente x,puede escribirse como sigue:

a0 (x) y(n)+a1 (x) y

(n�1)+ � � �+an�1 (x) y0+an (x) y =g (x) (1)

Donde g (x) y los coe�cientes a0 (x) ; a1(x); :::; an�1(x); an(x) dependensolamente de la variable x y no de y.

Ejemplos.

a).dy

dx= 3x E.D.O. lineal de primer orden.

b).d2y

dt2+ sen(t+ 1)y = C E.D.O. lineal de segundo orden.

c). y00 + y0 + ky = 0 E.D.O. lineal de segundo orden, donde k es unaconstante.

d). y{v � y = e�x. E.D.O. lineal de cuarto orden no lineal.Ecuación Diferencial No Lineal.Una ecuación diferencial ordinaria que no es de la forma (1), se llama

ecuación diferencial ordinaria no líneal.

Ejemplos.

a). y0 = ey . E.D.O. no lineal de primer orden.

3

b).d3y

dx3+ cos(y + 1)y = 0 E.D.O. de tercer orden no lineal.

c). (y0)2 + x = 0 E.D.O. de primer orden no lineal.

d). y000 � y0 = cos(x+ y) E.D.O. de tercer orden.

Ejemplos.Clasi�car cada una de las siguientes ecuaciones diferenciales de acuerdo al

tipo, orden y linealidad.

a).d3t

dy3+ y2 = C E.D.O. de tercer orden, no líneal.

b). (y00)2 +d2y

dx2= x2 + senx E.D.O. de segundo orden, no lineal.

c).@2u

@t2+@2v

@w2= t E.D.P. de segundo orden.

d). exd2y

dx2+ (lnx)y = 0 E.D.O. de segundo orden, lineal.

e). yiv + ky = ln (x+ y) E.D.O. de cuarto orden, no lineal, k, es unaconstante.

Solución de Una Ecuación Diferencial.Una solución de una ecuación diferencial es cualquier función f , de�nida en

un intervalo I, tal que, sí sustituida en dicha ecuación la reduce a una identidaden el intervalo especi�cado. El intervalo I puede representar: un intervaloabierto (a; b), cerrado [a; b], o in�nito (0;1), etc.

Ejemplos.1. La función y = xex es una solución de la ecuación diferencial lineal de

segundo orden

y00 + 2y0 + y = 0,

de�nida en el intervalo (�1;1).Solución.Las derivadas de la función y = xex son:

y0 = ex + xex

y00 = ex + ex + xex = 2ex + xex

4

Sustituyendo estas expresiones en la ecuación dada, tenemos

y00 � 2y0 + y = 2ex + xex � 2 (ex + xex) + xex= 2ex + xex � 2ex � 2xex + xex

=0:

2. La función y =senx

xes una solución de la ecuación lineal de primer

orden

xy0 + y = cosx,

de�nida en el intervalo de (0;1) :Solución.La derivada de la función y =

senx

xes:

y0 =x cosx� senx

x2.

Sustituyendo, en la ecuación inicial, tenemos.

xy0 + y = x

�x cosx� senx

x2

�+senx

x

=x cosx� senx

x+senx

x

=x cosx

x� senx

x+senx

x

= cosx:

3. Veri�car que la función y = 232x2, es una solución de la ecuación defer-

encial no lineal,

y0 � 4 (xy)13 = 0,

de�nida en el intervalo (0;1).Solución.La derivada de la función y = 2

32x2 es:

5

y0 = 232 2x = 2

52x

Sustituyendo y0 en la ecuación inicial, tenemos

y0 � 4 (xy)13 = 2

52x� 4

hx�232x2

�i 13

= 252x� 4

�212x�= 0

= 252x� 22 � 2 12x

= 0:

4. Veri�car que la función y =x4

16es una solución de la ecuación no lineal.

y0 = xy12 ,

de�nida en el intervalo (�1;1).

Solución.

La derivada de la función y =x4

16es:

y0 =x3

4.

Reemplazando la derivada de la función en la encuación dada, tenemos

y0 � xy 12 = x3

4� x

�x4

16

� 12

=x3

4� x

�x4

42

� 12

=x3

4� x

�x2

4

=x3

4� x

3

4

= 0:

6

Soluciones Explicitas e Implicitas.Una ecuación diferencial puede tener como soluciones explicítas e implicítas.Soluciones Explícitas.Se llama solución explicita de una ecuación diferencial a una función que se

puede escribir en la forma y = f (x), en el intervalo I.

Ejemplo.Las funciones de los ejemplos anteriores y = xex, y =

senx

x, y = 2

32x2 son

soluciones explícitas de sus correspondientes ecuaciones diferenciales.Soluciones Implícitas.Se llama solución implícita de una ecuación diferencial a la función G(x; y),

si de�ne una o más soluciones explícitas en el intervalo I.

Ejemplo.Veri�car que la función x2+y2 = 16 es una solución implicita de la ecuación

diferencial de primer orden y0 = �xyen el intervalo (�4; 4) :

Solución.

La función x2 + y2 = 16, derivamos implicitamente.

2x+ 2yy0 = 16.

Despejando y0 tenemos

y0 = �xy,

que es idéntica a la ecuación inicial. Por otra parte, la función x2 + y2 = 16de�ne dos funciones implicítas

y =p16� x2 y y = �

p16� x2.

En consecuencia, la función x2 + y2 = 16 es una solución implícita de laecuación y0 = �x

y.

Ejemplo.Muestre que xy+exy = 0 es una solución implicita de la ecuación diferencial

(x+ xexy) y0 + y + yexy = 0.

Solución.

Derivamos implicitamente la función xy + exy = 0, tenemos

7

y + xy0 + yexy + xy0exy = 0.

Luego

(x+ xexy) y0 + y + yexy = 0.

Se entiende también por solución implícita, cuando la solución no se puedeexpresar en la forma y = f (x).

Familia de Soluciones.Una ecuación diferencial tiene generalmente un número in�nito de soluciones

es decir una familia de soluciones con un parámetro llamado constante arbitrariaC.

Ejemplos.1. La ecuación diferencial de primer orden

y0 = 2y

x, x 6= 0 ,

tiene por solución la familia de parabolas, y = Cx2, donde C, es una con-stante.

Solución.Primeramente veri�quemos que la función y = Cx2 es una solución de la

ecuación planteada. La primera derivada es y0 = 2Cx. Sustituyendo en laecuación, resulta

y0 � 2yx= 2Cx� 2Cx

2

x= 2Cx� 2Cx = 0:

Dando a C, distintos valores generamos la familia de parábolas.

2. La función y =C

x+ 1, donde C es una constante, es una solución de la

ecuación lineal de primer orden

xy0 + y = 1,

en el intervalo (0;1).Solución.

Donde y0 = �Cx�2 = � Cx2, sustituyendo en la ecuación inicial, veri�camos

8

Parabolas

1:pdf

Hiperbolas

2:pdf

9

xy0 + y = x

�� Cx2

�+C

x+ 1 = �Cx

x2+C

x+ 1 = �C

x+C

x+ 1 = 1.

Dando a C distintos valores generamos la falimia de soluciones.

3. Las funciones y1 = C1 sen 2x y y2 = C2 cos 2x, donde C1 y C2 son dosconstantes, son soluciones de la ecuación diferencial de segundo orden y00+4y =0:

Solución.Las derivadas de la función y1 = C1 sen 2x son:

y01 = 2C1 cos 2x

y001 = �4C1 sen 2x

Vemos que:

y00 + 4y = �4C1 sen 2x+ 4C1 sen 2x = 0.

Por otra parte, las derivadas de la función y2 = C2 cos 2x son:

y02 = �2C2 sen 2x

y002 = �4C2 cos 2x

Así que:

y00 + 4y = �4C1 cos 2x+ 4C1 cos 2x = 0.

La suma de estas soluciones, y = C1 sen 2x + C2 cos 2x, también es unasolución de la ecuación dada. La prueba de esta solución es en forma análoga.

La grá�ca de las funciones y1 = C1 sen 2x, y2 = C2 cos 2x. y y = C1 sen 2x+C2 cos 2x para C1 = C2 = 1, se muestran en las �guras de abajo.

10

2 1 1 2 3 4

2

1

1

2Sen 2 x

2 1 1 2 3 4

2

1

1

2cos 2 x

2 1 1 2 3 4

2

1

1

2cos 2 x Sen 2 x

TEMA No 2

ECUACIONES DIFERENCIALES DE PRIMERORDEN

En general una ecuación diferencial de primer orden se puede escribir de lasiguiente manera:

F (x; y; y0) = 0 (Forma Implicíta)

Sí en está ecuación es posible despejar y0, se tiene:

dy

dx= f (x; y), o bien y0 = f (x; y) (Forma Explícita)

representa una ecuación diferencial de primer orden, donde f es una funciónde dos variables x y y.Problema de Valor Inicial.El problema de valor inicial de una ecuación diferencial de primer orden

11

y0 = f (x; y)

signi�ca, encontrar una solución de la ecuación diferencial en un intervalo Ique satisfaga la condición.

y (x0) = y0,

donde x0 es un número en el intervalo I, y y0 es un número real.La relación y (x0) = y0, se llama "condición inicial" o "valor inicial" de la

ecuación dada. Gra�camente signi�ca:

Soluciones de la ecuación diferencial.

Ejemplo.Consideremos nuevamente la ecuación

y0 =2y

x

y la condición inicial y (1) = 1.Solución.Representa un problema de valor inicial, sabemos que la ecuación tiene una

familia de soluciones y = Cx2.Para obtener una solución particular, que satisfaga la condición y(1) = 1, es

necesario sustituir x = 1 y y = 1 en la familia de soluciones: y = Cx2.Sí: x = 1 y y = 1 =) y = Cx2

1 = C(1)2 =) C = 1.De modo que: y = x2, es una solución particular de la ecuación dada, que

es una solución única. Geométricamente, esto es

­4 ­2 0 2 4

5

10

15

x

y

12

Teorema de Existencia y Unicidad de una Solución (T.E.U.)Sí en el problema de valor inicial y0 = f (x; y) y y(x0) = y0, la función

f (x; y) y su derivada parcial@f

@yrespecto de y son continuas en una región

rectangular R, en el plano xy y que contiene el punto (x0; y0), entonces existeuna y sóla una solución y = f(x) de la ecuación diferencial dada que satisfacela condición y = y0 cuando x = x0.

Ejemplo.Aplicar el T.E.U. para determinar si existe una solución única a el problema

de valor inicial.

y0 = x3 + y2, y(1) = 2.

Solución.Vemos que f(x; y) = x3 + y2 y

@f

@y= 2y, son continuas en cualquier región

rectangular que contenga al punto (1; 2). Por tanto el T.E.U. garantiza unasolución única al problema de valor inicial en una región R que pasa por elpunto (1; 2).

Ejemplo.Dado el problema de valor inicial.

y0 =3

2y23 y y(2) = 0,

veri�car si se cumple el T.E.U.Solución.Donde f(x; y) =

3

2y23 , su derivada parcial respecto de y es:

@f

@y=2

3

3

2y�

13 =

13py.

13

Desafortunadamente la@f

@yno es continua puesto que no está de�nida para

el punto y = 0, esto signi�ca que no existe una región rectangular R que contieneal punto (2; 0). Su restricción es y > 0.

Por tanto T.E.U. no garantiza una solución única al problema dado, puesno existe una región R por el cúal pase el punto (2; 0).

El problema de valor inicial, tiene una solución y = (x� 2)3, también y = 0es una solución, una tercera solución también lo es

y =

�(x� 2)3 sí: x > 20 sí: x < 2

1 2 3 4 5­5

0

5

10

15

20

25

x

y

Ejemplo.Dado el problema de valor inicial y0 = xy

12 , y(0) = 0. Veri�car si se cumple

el T.E.U.Solución.Donde: f (x; y) = xy

12 , su derivada parcial para está función, resulta:

@f

@y=1

2xy�

12 =

x

2py.

Lamentablemente@f

@yno es continua, tiene restricciones, puesto que no está

de�nida en el punto y = 0, esto establece que no existe una región rectángularR, que contenga al punto (0; 0). El T.E.U. no garantiza una solución única a elproblema de valor inicial. En la grá�ca se puede observar, que tiene al menos

dos soluciones cuyas grá�cas pasan por (0; 0), siendo y = 0 y y =x4

16.

Pero, si el semiplano y > 0, existe una solución única en cualquier región

rectángular R. Así, por ejemplo para cualquier punto (x0; y0), y > 0. Para

14

(2; 1), existe una solución única, esto signi�ca que existe una región rectangularR, que contiene al punto (0; 0).

ECUACIONES SEPARABLESSi la ecuación diferencial de primer orden.

dy

dx= f (x; y),

se puede escribir en la forma diferencial.

M (x) dx+N (y) dy = 0

o bien

dy

dx= �M (x)

N (y).

se llama ecuación separable o ecuaciones de variables separables.Método Solución.La ecuación diferencial separable.

dy

dx= �M (x)

N (y)(1)

también se puede escribir

N (y)dy

dx= �M (x),

dondedy

dx= y0 = f 0 (x).

Supongase que y = f (x) es la solución de (1), entonces

N(f (x))f 0 (x) = �M (x).

Integrando respecto de x, tenemos.RN(f (x))f 0 (x) dx = �

RM (x) dx+ C.

Como dy = f 0 (x) dx y y = f (x).

15

RN (y) dy = �

RM (x) dx+ C

ó bien RM (x) dx+

RN (y) dy = C.

En el caso particular sí N (y) = �1, en la ecuación (1) se reduce

dy

dx=M (x)

ó bien

dy =M (x) dx

Integrando Rdy =

RM (x) dx+ C.

Por tanto,

y =RM(x)dx+ C

obtenemos una solución explicita.

Ejemplo.Resolver la ecuación diferencial

dy

dx=y � 1x+ 3

.

Solución.La ecuación es de variables separables, pues

dy

y � 1 =dx

x+ 3.

Integrando Z1

y � 1dy =Z

1

x+ 3dx+ C1

ln jy � 1j = ln jx+ 3j+ ln jC1j,

donde C1 = ln jCj, luego aplicando la propiedad de logaritmos, tenemos

ln jy � 1j = ln jC (x+ 3)jy � 1 = C(x+ 3).

Obtenemos una

y = C(x+ 3) + 1

16

solución explicita.Ejemplo.Resolover la ecuación

dy

dx=

y2(x� 1)x2(2y3 � y) .

Solución.Separando variables, tenemos

dy

dx=

y2

2y3 � y �x� 1x2

2y3 � yy2

dy =x� 1x2

dx.

Integrando, tenemos:Z2y3 � yy2

dy =

Zx� 1x2

dx+ C

Z2y3

y2dy �

Zy

y2dy =

Zx

x2dx�

Z1

x2dx+ C

Z2ydy �

Z1

ydy =

Z1

xdx�

Z1

x2dx+ C

2

Zydy �

Z1

ydy =

Z1

xdx�

Zx�2dx+ C

2

�y2

2

�� ln jyj = ln jxj �

�x�1

�1

�+ C

y2 � ln jyj = ln jxj+ 1

x+ C

y2 � ln jxj � ln jyj � 1x= C

y2 � (ln jxj+ ln jyj)� 1x= C.

Obtenemos una

y2 � 1x� ln jxyj = C

solución implicita.

Ejemplo.Resolver la ecuación.

17

xp1 + y2dx+ y

p1 + x2dy = 0.

Solución.Separando variables se tiene.

yp1 + x2dy = �x

p1 + y2dx

yp1 + y2

dy = � xp1 + x2

dx.

Integrando Zyp1 + y2

dy=-Z

xp1 + x2

dx+ C.

Aplicando un cambio de variable a ambos integrales, tenemos.Si: u = 1 + y2 ) du = 2ydy y Sí: z = 1 + x2 ) dz = 2xdx

du

2= ydy.

dz

2= xdx .

1

2

Zdupu= � 1

2

Zdzpz+ C

1

2

Z1

u12

du = � 1

2

Z1

z12

dz + C

1

2

Zu�

12 du = � 1

2

Zz�

12 dz + C

1

2

u12

12

= � 1

2

z12

12

+ C

u12 = � z 12 + C.

Sí: u = 1 + y2 y z = 1 + x2, tenemos

(1 + y2)12 = �

�1 + x2

� 12 + C

�1 + x2

� 12 +

�1 + y2

� 12 = C.

Así, obtenemos una solución implícitap1 + x2 +

p1 + y2 = C.

18

Ejemplo.Resolver la ecuación.

y0 senx = y ln y,

sujeto a la condición inicial y��2

�= e.

Solución.Se trata de un problema de valor inicial, separando variables, tendremos

dy

dxsenx = y ln y

dy senx = y ln ydx

dy

y ln y=

dx

senx.

Integrando Z1

y ln ydy =

Z1

senxdx+ C1.

Sí: u = ln y ) du =1

ydy:

Zdu

u=

Z1

senxdx+ C1

ln juj = ln���tan x

2

���+ ln jCj.Donde C1 = ln jCj,

ln jln yj = ln���C � tan x

2

���ln jyj = C tan x

2.

Por tanto, la solución general es:

y = eC tan

x

2 .

Sustituyendo la condición inicial x =�

2y y = e, resulta.

19

e = eC tan

�2

2

e = eC .

Aplicando logaritmo neperiano, tenemos

ln e = C ln e ó e1 = eC

1 = C C = 1.

Por tanto, la solución pariticular es

y = etanx

2 .

Ejemplo.Resolver la ecuación.

y0 = 6x(y � 1) 23 .

Solución.Separando las variables, tenemos

dy

dx= 6x(y � 1) 23

dy

(y � 1) 23= 6xdx.

Integrando, tenemos Zdy

(y � 1) 23=

Z6xdx+ C.

Sí: u = y � 1) du = dy.Z1

u23

du = 6

Zxdx+ C

Zu�

23 du = 6

Zxdx+ C

u13

13

= 6x2

2+ C

3u13 = 3x2 + C.

20

Sustituimos u = y � 1, tenemos:

3 (y � 1)13 = 3x2 + C

(y � 1)13 =

3x2 + C

3.

Elevando a la tercera potencia a ambos miembros, tenemos:h(y � 1)

13

i3=

�3x2 + C

3

�3y � 1 =

�3x2

3+C

3

�3y =

�x2 +

C

3

�3+ 1

y =�x2�3+ 3

�x2�2 C3+ 3x2

�C

3

�2+

�C

3

�3+ 1

y = x6 + Cx4 +C2x2

3+C3

27+ 1.

Ejemplo.Resolver el problema de valor inicial

e�y (1 + y0) = 1, con y(0) = �1.

Solución.La ecuación escribimos de la siguiente forma

1 + y0 =1

e�y

y0 = ey � 1

dy

dx= ey � 1.

Separando variables, tenemos:

dy

ey � 1 = dx.

Integrando Zdy

ey � 1 =Zdx+ C

Z1

ey � 1dy =Zdx+ C.

21

Multiplicando y dividiendo por e�y, la última ecuación, tenemosZ1

ey � 1

�e�y

e�y

�dy =

Zdx+ C

Ze�y

1� e�y dy =Zdx+ C.

Sí: u = 1� e�y ) du = e�ydy, se tieneZdu

u=

Zdx+ C

ln juj = x+ C

u = ex+C

1� e�y = ex+C .

Así, obtenemos la solución general

e�y = 1� exC.

Utilizando la condición x = 0 y y = �1, tendremos

e�(�1) = 1� e0C

e = 1� C

C = 1� e.

Por tanto, una solución particular es

e�y = (1� ex) (1� e).

ECUACIONES HOMOGÉNEASFunción Homogénea.La función f(x; y), se llaman función homogénea de grado n, respecto de

las variables x y y, si la función f (x; y) tiene la propiedad de que

f (tx; ty) = tnf(x; y); 8, n y t 2 R.

22

Ejemplo.Determinar si las funciones son homogéneas o no, e indicar el grado de la

homogeneidad.

a). f(x; y) = x2 � 3xy + y2:

f(tx; ty) = (tx)2�3(tx)(ty)+(ty)2 = t2x2�3t2xy+t2y2 = t2(x2�3xy+y2) =t2f(x; y).

La función es homogénea de grado dos.

b). f(x; y) = 3px2 � 16y2.

f(tx; ty) = 3p(tx)2 � 16(ty)2 = 3

pt2x2 � 16t2y2 = 3

pt2(x2 � 16y2) = t 23 3

px2 � 16y2 =

t23 f(xy).

La función es homogénea de grado 23 .

c). f(x; y) =3x2 + y2

x2 � 2y2

f(tx; ty) =3(tx)2 + (ty)2

(tx)2 � 2(ty)2 =t2(3x2 + y)2

t2(x2 � 2y)2 = t0 (3x

2 + y)2

(x2 � 2y)2 = t0f(x; y).

La función es homogénea de grado cero.

d) f(x; y) =4y

x+1

2.

f(tx; ty) =4(ty)

(tx)+1

2= t0

�4y

x+1

2

�= t0f(x; y).

La función es homogénea de grado cero.

e) f(x; y) = x3 + 3xy2 � y3 + 2.f(tx; ty) = (tx)3+3(tx)(ty)2�(ty)3+2 = t3(x3+3xy2�y3+2t�3) 6= t3f(x; y).

La función no es homogéneaEcuación Homogénea.Una ecuacion diferencial de primer orden

dy

dx= f(x; y), (1)

se llama ecuación homogénea, si la función f(x; y) es homogénea de gradocero respecto de x y y.Método de Solución.Por hipótesis

23

f(tx; ty) = t0f(x; y) = f(x; y):

Haciendo t =1

x, tenemos

f(x; y) = f�1;y

x

�. (2)

De la ecuación (1) y (2), obtenemos.

dy

dx= f(1;

y

x). (3)

Ahora efectuamos la sustitición

u =y

xo bien y = ux,

derivando respecto de x, se tiene

dy

dx= u+ x

du

dx. (4)

De la ecuación (3) y (4), tenemos

u+ xdu

dx= f(1; u),

que es una ecuación de variables separables

xdu

dx= f(1; u)� u

du

f(1; u)� u =du

x,

integrando, obtenemosZdu

f(1; u)� u =Zdu

x+ C.

Finalamente, obtenemos la solución sustituyendo el valor de u pory

x.

Ejemplo.Resolver la ecuación

24

dy

dx=2xy + y2

x2

Solución.La ecuación escribimos en forma (3)

dy

dx=2y

x+�yx

�2.

La ecuación es homogénea de grado cero. Sustituyendo u =y

xydy

dx=

u+ xdu

dxen la ecuación, resulta

u+ xdu

dx= 2u+ u2.

Separando las variables, se tiene

xdu

dx= u+ u2

du

u+ u2=dx

x.

Integrando Zdu

u(1 + u)=

Zdx

x+ C1.

Integrando por fracciones parciales.Z �A

u� B

1 + u

�du =

Z1

xdx+ C1. De donde: A = 1, B = �1.

Z �1

u� 1

1 + u

�du =

R 1xdx+ C1, donde C1 = ln jCj.

ln juj � ln j1 + uj = ln jxj+ ln jCj

ln

���� u

1 + u

���� = ln jCxjy

x

1 +y

x

= Cx

y

x+ y= Cx.

25

Por tanto, la solución general es

y =Cx2

1� Cx .

Ejemplo.Resolver la ecuación

4x� 3y + dy

dx(2y � 3x) = 0.

Solución.La ecuación escribimos en la forma

dy

dx=3y � 4x2y � 3x .

Multiplicado numerador y denominador por1

x, obtenemos

dy

dx=3y

x� 4

2y

x� 3

,

una ecuación homogénea de grado cero. Sustituyendo u =y

xydy

dx= u+x

du

dx

en la última ecuación, resulta

u+ xdu

dx=3u� 42u� 3 .

Separando las variables, se tiene

xdu

dx=3u� 42u� 3 � u

xdu

dx=3u� 4� 2u2 + 3u

2u� 3

xdu

dx=�2u2 + 6u� 4

2u� 3

2u� 3�2u2 + 6u� 4du =

dx

x.

Integrando

26

�12

Z2u� 3

u2 � 3u+ 2du =Zdx

x+ C1

�12ln��u2 � 3u+ 2�� = ln jxj+ ln jC1j

ln��u2 � 3u+ 2��� 1

2 = ln jC1 � xj.

Sustituyendo u =y

x, obtenemos

"�y2

x2� 3y

x+ 2

�� 12

#�2= [C1x]

�2

y2 � 3xy + 2x2x2

= [C1x]�2

y2 � 3xy + 2x2 = C�21 x�2x2

donde C = C�21 . Por tanto, la solución general es

y2 � 3xy + 2x2 = C .

Ejemplo.Resolver la ecuación

dy

dx=x2 sec

�yx

�+ y2

xy,

dada la condición inicial y(1) = 0.Solución.La ecuación escribimos en forma (3)

dy

dx=x

ysec

y

x+y

x

dy

dx=�yx

��1sec�yx

�+y

x.

La ecuación es homogénea de grado cero. Sustituyendo u =y

xydy

dx=

u+ xdu

dxen la última ecuación, resulta

27

u+ xdu

dx= u�1 secu+ u.

Separando las variables, se tiene

xdu

dx= u�1 secu

du

u�1 secu=dx

x

u cosu du =dx

x.

Integrando Zu cosu du =

Zdx

x+ C.

Integrando por partes, tenemosZzdv = zv �

Zvdz

z = u, dv =

Zcosudu,

dz = du.v = senu.Luego

u senu�Zsenu du = ln jxj+ C

u senu+ cosu = ln jxj+ C.

Sustituyendo u =y

x, obtenemos la solución general

y

xsen

y

x+ cos

y

x= ln jxj+ C.

Utilizando la condición x = 1 y y = 0, obtenemos

28

1 = C.

Por tanto, la solución particular es

y

xsen

y

x+ cos

y

x= ln jxj+ 1.

ECUACIONES EXACTASConsideremos la ecuación diferencial

M(x; y)dx+N(x; y)dy = 0. (1)

Supóngase que existe una función f(x; y), tal que

@f

@x=M(x; y) ;

@f

@y= N(x; y).

Entonces la ecuación (1), se puede escribir como:

@f

@xdx+

@f

@ydy = 0

o bien

d [f(x; y)] = 0

y su solucion general es f(x; y) = C.

A la expresión

M(x; y)dx+N(x; y)dy

se llama diferencial exacta y (1) se llama ecuación exacta:Ejemplo.Muestre que la ecuación diferencial

2xy2dx+ 3x2y2dy = 0 ,

es exacta y determine las soluciones.Solución.Vemos que el primer miembro de la ecuación es la derivada de la función

f(x; y) = x2y3. Entonces la ecuación dada podemos escribir

29

d�x2y3

�= 0,

En consecuencia, la solución general está dada implicitamente por x2y3 = C,y explícitamente por y = k x�

23 .

En el ejemplo, planteado no fue difícil veri�car la exactitud de la ecuacióndada, pero en ecuaciones más difíciles no es tan sencillo determinar f(x; y) porsimple inspección. A continuación planteamos un teorema importante, paradeterminar la exactitud de una ecuación diferencial.Teorema (Criterio Para Una Ecuación Exacta).Supongase que las funciones M(x; y) y N(x; y), y sus derivadas parciales

son continuas en una región rectangular R. Entonces una condición necesariay su�ciente para que

M(x; y)dx+N(x; y)dy = 0,

sea exacta en R, es si y sólo si

@M

@y=@N

@x, 8 (x; y) 2 R.

Demostración. El teorema demostraremos en dos partes:Condición Necesaria:Para simpli�car la condición necesaria, consideremos que existe una función

f(x; y) tal que se cumple

M(x; y)dx+N(x; y)dy =@f

@xdx+

@f

@ydy

donde

M(x; y) =@f

@xy N(x; y) =

@f

@y.

Derivando parcialmente la primera ecuación respecto de y, tenemos

@M

@y=@f

@y

�@f

@x

�=@2f

@y@x=@

@x

�@f

@y

�=@N

@x, 8 (x; y) 2 R.

30

Condición Su�ciente:Esta condición demostramos en la siguiente sección, que consiste en con-

struir una función f(x; y), en realidad este hecho no es más que el método desolución.

Método de Solución.Sea la ecuación

M(x; y)dx+N(x; y)dy = 0

tal que

@M

@y=@N

@x;

donde@f

@x=M(x; y),

@f

@y= N(x; y).

Integrando la primera ecuación con respecto de x mientras y se mantienecomo constante, se tiene.

f(x; y) =RM(x; y)dx+ g(y). (2)

La función arbitraria g(y) es la "constante de integración" . Derivado (2)

respecto de y, y teniendo en cuenta que@f

@y= N(x; y) resulta.

@f

@y=@

@y

RM(x; y)dx+ g0(y) = N(x; y)

luego

g0(y) = N(x; y)� @

@y

RM(x; y)dx.

Integrando respecto de y, obtenemos

g(y) =R �N(x; y)� @

@y

RM(x; y)dx

�dy,

�nalmente obtenemos la solución general, sustituyendo el valor de g(y)en (2), la solución es de la forma:

31

f(x; y) = C.

Por otra parte si se toma la ecuación

@f

@y= N(x; y),

integrando respecto de y, y derivando el resultado, obtenemos en formaanáloga

f(x; y) =RN(x; y)dy + h(x) y h(x) =

R �M(x; y)� @

@x

RN(x; y)dy

�dx .

Ejemplo.Resolver la ecuación

x(2x2 + y2)dx+ y(x2 + 2y2)dy = 0.

Solución.Probaremos si esta ecuación dada es exacta, siendo

M(x; y) = x(2x2 + y2), N(x; y) = y(x2 + 2y2).

Derivando

@M

@y= 2xy,

@N

@x= 2xy

como

@M

@y=@N

@x.

La ecuación dada es exacta, donde

@f

@x=M(x; y) = x(2x2 + y2) y

@f

@y= N(x; y) = y(x2 + 2y2).

Integrando la primera ecuación respecto de x tenemosf(x; y) =R

(2x3 + xy2)dx+ g(y)

f(x; y) =1

2x4 +

x2

2y2 + g(y). (3)

32

Derivando (3) respecto de y, y como@f

@y= N(x; y), resulta

@f

@y= x2y + g0(y) = y(x2 + 2y2).

Luego

g0(y) = 2y3.

Integrando

g(y) =R2y3dy =

1

2y4.

Sustituyendo el valor de g(y) en (3), obtenemos

f(x; y) =1

2x4 +

1

2x2y2 +

1

2y4.

Multiplicando la ecuación anterior por 2, por tanto la solución general es:

x4 + x2y2 + y4 = C.

EjemploResolver la ecuación

(y + y cosxy)dx+ (x+ x cosxy)dy = 0.

Solución.Siendo

M(x; y) = y + y cosxy y N(x; y) = x+ x cosxy.

Probemos si la ecuación inicial es exacta

@M

@y= 1 + cosxy � xy senxy, @N

@x= 1 + cosxy � xy senxy,

como son iguales

@M

@y=@N

@x.

33

La ecuación dada es exacta, donde

@f

@x=M(x; y) = y + y cosxy y

@f

@y= N(x; y) = x+ x cosxy.

Integrando la primera ecuación respecto de x tenemos

f(x; y) =R(y + y cosxy)dx+ g(y)

f(x; y) = xy + senxy + g(y). (4)

Derivando (4) respecto de y y como@f

@y= N(x; y), resulta

@f

@y= x+ x cosxy + g0(y) = x+ x cosxy

g0(y) = 0.

Integrando

g(y) = C1.

Sustituyendo el valor de g(y) en (4), obtenemos

f(x; y) = xy + senxy + C1.

Por lo tanto, la solución general es:

xy + senxy = C.

FACTOR INTEGRANTEConsideremos que la ecuación

M (x; y) dx+N (x; y) dy = 0, (1)

no es exacta. Muchas veces se logra hallar una función � (x; y), tal quemultiplicada todos los términos de la ecuación (1), por esta función se convierteen una ecuación diferencial exacta. La función � (x; y), se llama factor integrante

de la ecuación (1).Para determinar un factor integrante de (1), multiplicamos los dos miembros

de (1), por la función �, esto es

34

�Mdy+ �Ndx = 0.

Esta ecuación será exacta si cumple la siguiente condición:

@(�M)

@y=@(�N)

@x.

Aplicando la regla del producto de la derivada, se tiene

�@M

@y+M

@�

@y= �

@N

@x+N

@�

@x

1

�N@�

@x�M @�

@y

�=@M

@y� @N@x. (2)

Supóngase que la ecuación (1), admite un factor integrante que depende

solamente de x, esto es, � = � (x), entonces@�

@y= 0 y

@�

@x=d�

dx, de modo que

(2) se reduce a :

1

d�

dx=

@M

@y� @N@x

N

o bien

d�

�=

0BB@@M

@y� @N@x

N

1CCA dx.Hacemos

g (x) =

@M

@y� @N@x

N.

Siendo

d�

�= g(x)dx.

Integrando

35

Z1

�d� =

Zg(x)dx

ln j�j =Zg(x)dx.

En consecuencia, el factor integrante de (1), que depende sólo de x es:

�(x) = eRg(x)dx.

En forma análoga podemos obtener, el factor integrante � de (1) que dependesolamente de y, esto es � = �(y), siendo

h(y) =

@N

@x� @M@y

M

Por tanto, el factor integrante de la ecuación (1) que depende solamente dey es:

�(y) = eRg(y)dy.

Ejemplo.Resolver la ecuación�

2x2y + 2y + 5�dx+

�2x3 + 2x

�dy = 0. (4)

Solución.La ecuación dada ni es separable, ni homogénea, veamos si es exacta. Donde:

M(x; y) = 2x2y + 2y + 5 y N(x; y) = 2x3 + 2x

@M

@y= 2x2 + 2 ,

@N

@x= 6x2 + 2.

Como

@M

@y6= @M

@y,

La ecuación no es exacta. Ahora veamos si (4) admite un factor integranteque depende sólo de x, esto es:

36

g(x) =2x2 + 2� 6x2 � 2

2x3 + 2x=

�4x22x (x2 + 1)

=�2x

(x2 + 1).

Luego, el factor integrante es

�(x) = e�

Z2x

x2+1dx

= e� lnjx2+1j = elnjx

2+1j�1 =�x2 + 1

��1

�(x) =1

x2 + 1.

Multiplicando la ecuación (4), por este factor integrante se tiene:�2x2y + 2y + 5

x2 + 1

�dx+

�2x3 + 2x

x2 + 1

�dy = 0

�2y(x2 + 1)

x2 + 1+

5

x2 + 1

�dx+

2x(x2 + 1)

x2 + 1dy = 0

�2y +

5

x2 + 1

�dx+ 2xdy = 0. (5)

Donde:

M(x; y) = 2y +5

x2 + 1y N(x; y) = 2x

@M

@y= 2 =

@N

@x.

Esto muestra que la ecuación (5) es exacta. De manera que:

@f

@x= 2y +

5

x2 + 1;

@f

@y= 2x.

Integrando la primera ecuación respecto de x, tenemos:

f(x; y) =

Z �2y +

5

x2 + 1

�dx+ g(y) = 2xy + 5arctanx+ g(y).

Derivando f(x; y) respecto de y, y como@f

@x= N(x; y) resulta :

37

@f

@y= 2x+ g0(y) = 2x

g0(y) = 0.

Integramos

g(y) = C.

Por tanto, la solución general es:

2xy + 5arctanx+ C = 0.

Ejemplo.Resolver la ecuación �

y + xy2�dx�xdy = 0. (6)

Solución.Donde:

M(x; y) = y + xy2 y N(x; y) = �x

@M

@y= 1 + 2xy

@N

@x= �1

@M

@y6= @N

@x.

La ecuación dada no es exacta. Veamos si (6) admite un factor integranterespecto de x

g(x) =

@M

@y� @N@x

N=1 + 2xy + 1

�x =2xy + 2

�x =�2(xy + 1)

x,

esta función no depende solamente de x. Ahora veamos si admite un factorintegrante respecto de y

h(y) =

@N

@x� @M@y

M=�1� 1� 2xyy + xy2

=�2� 2xyy + xy2

=�2� 2xyy + xy2

=

�2(1 + xy)y(1 + xy)

= �2y

38

La función h(y) es solamente de y. El factor integrante es�(y) = e

Rh(y)dy = e�2

R1y dy = e�2 lnjyj =

elnjyj�2

�(y) =1

y2.

Multiplicamos la ecuación (6) por el factor integrante

(y + xy2)

y2dx� x

y2dy = 0,

o bien �1

y+ x

�dx� x

y2dy = 0. (7)

Donde:

M(x; y) =1

y+ x y N(x; y) = � x

y2

@M

@y= � 1

y2=@N

@x.

La ecuación (7) es exacta. De manera que:

@f

@x=1

y+ x;

@f

@y= � x

y2.

Integrando la primera ecuación respecto de x, tenemos

f(x; y) =

Z �1

y+ x

�dx+ g(y) =

x

y+x2

2+ g(y).

Derivando f(x; y) respecto de y, y como@f

@y= N(x; y), resulta

@f

@y= � x

y2+ g0(y) = � x

y2

g0(y) = 0.

Integrando

g(y) = C.

39

Por tanto, la solución general es

x

y+x2

2= C.

ECUACIONES LINEALESUna ecuación de la forma

dx

dy+ P (x)y = Q(x), (1)

se llama ecuación diferencial lineal de primer orden donde P (x) y Q(x)funciones contínuas de x (o constantes) en el intervalo I.Método de Solución.La ecuación (1), escribimos en forma diferencial

[P (x)y �Q(x)] dx+ dy = 0. (2)

Veamos la exactitud de esta ecuación

@M

@y= P (x);

@N

@x= 0.

La ecuación (2), no es exacta. Sin embargo, se puede obtener sencillamenteun factor integrante �(x) que depende solamente de x. Efectuamos

[�(x)P (x)y � �(x)Q(x)] dx+ �(x)dy = 0.

La ecuación será exacta, si se cumple

@M

@y= �(x)P (x);

@N

@x=d�

dx.

De manera que

d�

dx= �(x)P (x).

Es una ecuación de variables separables

d�

�=P(x)dx

Integrando:

40

Zd�

�=ZP (x)dx

ln j�j =ZP (x)dx.

Por tanto el factor integrante de la ecuación (1), es:

�(x) = eRP (x)dx.

Ahora multiplicaremos la ecuación (1), por este factor integrante

eRP (x)dx dy

dx+ e

RP (x)p(x)y = e

RP (x)Q(x).

El primer miembro de esta ecuación es la derivada del producto del factorintegrante y y:

d

dx

heRP (x) � y

i= e

RP (x)dxQ(x).

Integrando

eRP (x) � y =

ZeRP (x)dxQ(x)dx+ C.

Por tanto la solución general de (1), es:

y = e�RP (x)dx

�ZeRP (x)dx �Q(x)dx+ C

�.

Ejemplo.Resolver la ecuación �

x2 + 4� dydx+ xy = 0. (3)

Solución.La ecuación escribimos en la forma estándar

dx

dy+

x

x2 + 4y = 0. (4)

Siendo

P (x) =x

x2 + 4y Q(x) = 0.

41

El factor integrante es.

�(x) = eRP (x)dx = e

12

R2x

x2+4dx= e

12 lnjx2+4j = elnjx

2+4j 12 = (x2 + 4) 12 .

Luego multiplicamos la ecuacion (4), por el factor integrante

(x2 + 4)12dy

dx+x(x2 + 4)

12

x2 + 4y = 0

(x2 + 4)12dy

dx+ x(x2 + 4)�

12 y = 0.

Resulta

d

dx

h(x2 + 4)

12 yi= 0.

Integrando

(x2 + 4)12 y = C.

Por tanto la solución general es:

y =C

(x2 + 4)12

=Cpx2 + 4

.

Ejemplo.Resolver la ecuación:

dy

dx=x senx� y

x.

Solución.La ecuación escribimos en la forma de (1):

dy

dx+1

xy = senx (5)

La ecuación (5) es lineal, con P (x) =1

x; y Q(x) = senx.

El factor integrante es

�(x) = e

R 1xdx= elnjxj = x.

Ahora multiplicaremos (5) por el factor integrante

42

xdy

dx+ y = x senx.

De manera que

d

dx[xy] = x senx.

Integrando

xy =

Zx senxdx+ C = �x cosx+

Zcos dx+ C = �x cosx+ senx+ C.

u = x, dv = senxdx,

du = dx. v =

Zsenxdx,

v =� cosx .Por tanto, la solución general es:

y = � cosx+ senxx

+C

x; para x > 0.

Ejemplo.Resolver la ecuación

(cos2 x senx)dy + (y cos3 x� 1)dx = 0.

Solución.La ecuación escribimos en la forma de (1)

dy

dx+

cos3 x

cos2 x senxy =

1

cos2 x senx

dy

dx+cosx

senxy =

1

cos2 x senx

o bien

dx

dy+ (cotx) y =

1

cos2 x senx. (6)

La ecuación (6), es una ecuación lineal con P (x) = cotx. El factor integrantees:

�(x) = e

Zcot xdx

= elnjsen xj = senx.

43

Ahora multiplicamos (6) por el factor integrante

senxdy

dx+ senx

cosx

senxy =

senx

cos2 x senx

o bien

senxdy

dx+ (cosx) y =

1

cos2 x.

De manera que

d

dx[(senx) y] =

1

cos2 x.

Integrando

(senx) y =

Z1

cos2 x+ C = tanx+ C.

Por tanto la solución general es:

y = secx+ C cscx.

TEMA No 3

ECUACIONES DIFERENCIALES LINEALES DESEGUNDO ORDEN

INTRODUCCIÓNUna ecuación diferencial lineal de segundo orden en su forma canónica es de

la siguiente forma:

d2y

dx2+ P (x)

dy

dx+Q(x)y = R(x)

o bien:

y00 + P (x)y0 +Q(x)y = R(x) (1)

donde P (x), Q(x) y R(x) son funciones continuas sobre un intervalo I. SiR(x) 6= 0 en (1), se llama ecuación no homogénea. Si R(x) = 0 en (1), es de laforma:

44

y00 + P (x)y0 +Q(x)y = 0

se llama ecuación homogénea.

Ejemplo.

1. La ecuación y00 + x2y0 + xy =ex, es una ecuación de 2do. orden nohomogénea.

2. La ecuación: y00 + (senx)y0 + y = 0, es una ecuación homogénea.

Teorema (Teorema de Existencia y Unicidad de Una Solución Única).Supongase que P (x), Q(x) y R(x) son funciones continuas sobre el intervalo

I, y que contienen al punto x0:Entonces existe una solución única y solo unoy(x) de la ecuación (1) sobre el intervalo completo I y las condiciones iniciales

y(x0) = y0, y0(x0) = y00:

Ejemplo.Muestre que la función y = senx, es una solución de la ecuación homogénea

y00 + y = 0 que satisface los valores iniciales y(0) = 0, y0(0) = 1.

Solución.Veri�camos que y = senx, evidentemente es una solución de la ecuación

dada , además y = cosx, más en general y = C1 senx + C2 cosx, donde C1y C2, son constantes arbitrarias y también son soluciones. Pero simplementey = senx satisface los valores iniciales, puesto que: y = senx:Si: y(0) = 0 , entonces sen(0) = 0:Luego, si y = senx =) y0 = cosx:Si: y0(0) = 1 , entonces cos(0) = 1:Esto muestra que la función y = senx es la única solución de la ecuación

y00 + y = 0, que satisface las condiciones iniciales dadas.Teorema A.Sea y1(x) y y2(x) dos soluciones linealmente independientes de la ecuación

homogenea:

y00 + P (x)y0 +Q(x)y = 0. (2)

En un intervalo [a; b] entonces la combinación lineal

y = C1y1(x) + C2y2(x),

45

se llama "solución general" de la ecuación (2), para todo par de constantesC1 y C2.Demostración.Sea y(x), cualquier solución de la ecuación (2) sobre el intervalo [a; b]. Ahora

debemos probar que es posible hallar las constantes C1 y C2 tal que: y(x) =C1y1(x) + C2 y2(x); 8x 2 [a; b]. Todo queda completamente determinado porel valor y(x0) y el valor de su derivada y0(x0) en un único punto x0. Esto es:�

C1y1(x0) + C2y2(x0) = y(x0)C1y

01(x0) + C2y

02(x0) = y

0(x0):

Este sistema de ecuaciones tiene una solución única, para las constantes C1y C2, siempre que la determinante de los coe�cientes sea diferente de cero, esdecir: ���� y1(x0) y2(x0)

y01(x0) y02(x0)

���� = y1(x0)y02(x0)� y2(x0)y01(x0) 6= 0.La determinante de�nida por

W (y1,y2) =

���� y1 y2y01 y02

���� = y1y02 � y2y01,se conoce como el "wronskiano" de y1 y y2:

Corolario.Dos soluciones y1(x) y y2(x) de la ecuación homogénea

y00 + P (x)y0 +Q(x)y = 0,

en un intervalo [a; b], son linealmente independientes, si y sólo si, su wron-skiano es diferente de cero.

Ejemplo.Dada la ecuación lineal homogénea

y00 + 4y = 0.

a). Muestre que y1 = sen 2x y y2 = cos 2x son dos soluciones linealmenteindependientes.b). Hallar la solución general.c). Hallar la solución particular para los valores iniciales y(0) = 4 , y0(0) = 3.

Solución.a). Las soluciones y1 = sen 2x y y2 = cos 2x, serán linealmente independi-

entes si su wronskiano es diferente de cero, es decir:

46

W (y1, y2) =

���� sen 2x cos 2x2 cos 2x � 2 sen 2x

���� = �2 sen2 2x�2 cos2 2x = �2(sen2 2x+cos2 2x) = �2 6= 0.Esto muestra que evidentemente y1 y y2 son soluciones linealmente inde-

pendientes.b). La combinación lineal de las soluciones dadas es la solución general, esto

es

y = C1 sen 2x+ C2 cos 2x.

c). Para obtener la solución particular resolvemos el sistema:�C1 sen 2x+ C2 cos 2x = y;

2C1 cos 2x� 2C2 sen 2x = y01:

Aplicando las condiciones iniciales en el sistema, resulta�C1 sen 2(0) + C2 cos 2(0) = 4;

2C1 cos 2(0)� 2C2 sen 2(0) = 3:

De donde las soluciones son: C1 =3

2y C2 = 4. En consecuencia, la solución

particular buscada es:

y =3

2sen 2x+ 4 cos 2x.

REDUCCIÓN DE ORDEN.Para desarrollar este procedimiento, supongase que y1(x) es una solución no

nula conocida de la ecuación homogénea

y00 + P (x)y0 +Q(x)y = 0. (1)

De hecho Cy1(x), donde C es una constante, también es una solución de (1).Ahora la idea básica es sustituir C por una función desconocida digamos v(x)e intentar hallar v(x), tal que y2(x) = v(x)y1(x) sea la solución de la ecuación(1).Como suponemos que y2 = vy1 es una solución de la ecuación diferencial

(1), entonces debe cumplirse que

y002 + P (x)y02 +Q(x)y2 = 0. (2)

47

Las derivadas de y2 = vy1son

y02 = vy01 + v

0y1

y002 = vy001 + v

0y01 + v0y01 + v

00y1 = vy001 + 2v

0y01 + v00y1

Sustituyendo estas expresiones en (2), tenemos

(vy001 + 2v0y01 + v

00y1) + P (vy01 + v

0y1) +Qvy1 = 0,

reordenando los terminos queda

v (y001 + Py01 +Qy1) + y1v

00 + (2y01 + Py1)v0 = 0

como y1 es una solución de (1), entonces se reduce a

y1v00 + (2y01 + Py1)v

0 = 0.

Hacemos la sustitución u = v0, entonces se reduce a

y1u0 + (2y01 + Py1)u = 0.

una ecuación diferencial lineal de primer orden y de variables separables

u0

u= �2y

01

y1� P

Integrando: Z1

udu = �2

Zy01y1dx�

ZPdx

ln juj = �2 ln jy1j �ZPdx

u =elnjy1j�2e�

ZPdx

.

Como u = v0, se tiene

v0 = y�21 e�

ZPdx

.

Intengrando nuevamente, resulta

48

v(x) =

Z1

y21e�

ZP (x)dx

dx.

Por tanto la segunda solución linealmente independiente de la ecuación (1)es:

y2 = vy1

y2 = y1

Z1

y21e�

ZP (x)dx

dx.

Ejemplo.La función y1 = x2 es una solución de la ecuación

x2y00 � 3xy0 + 4y = 0,

determinar la segunda solución linealmente independiente y la solución gen-eral.Solución.Escribimos la ecuación en su forma canónica

y00 � 3xy0 +

4

x2y = 0.

Como P (x) = � 3x, la segunda solución viene dado por y2 = vy1. Deter-

minemos v(x):

v(x) =

Z1

[x2]2e

Z�3

xdx

dx =

Z1

x4e3

Z1xdx

dx =

Z1

x4e3 lnjxjdx =

Z1

x4x3dx =

Z1

xdx = ln jxj.

Luego la segunda solución es

y2 = x2 ln jxj.

Por tanto, la solución general es

y = C1y1(x) + C2y2(x)

y = C1x2 + C2x

2 ln jxj.

49

Ejemplo.La función y1 =

senxpxes una solucion de

x2y00 � xy0 +�x2 � 1

4

�y = 0, en el intervalo (0; �),

determine una segunda solución linealmente independiente.Solución.La ecuación en su forma canónica es:

y00 +1

xy0 +

�1� 1

4x2

�y = 0.

Como P (x) =1

xla segunda solución viene dado por y2 = vy1. Determinemos

v(x):

v(x) =

Z1�

senxpx

�2e�Z�1

xdx

dx =

Z1

sen2 x

x

e� lnjxjdx =Z

1

sen2 x

x

elnjxj�1dx =

Zx

sen2 xx�1dx =

Z1

sen2 xdx

=Rcsc2 xdx = � cotx.

Luego la segunda solución es

y2 = (� cotx)senxpx= � cosx

senx� senxp

x= �cosxp

x,

ó simplemente y2 =cosxpx.

Por tanto, la solución general es

y = C1senxpx+ C2

cosxpx.

Ejemplo.La función y1 = x3 es una solución de la ecuación

x2y00 � (2x2 + 6x)y0 + (6x+ 12)y = 0.

Hallar la segunda solución linealmente independiente y la solución general.Solución.La ecuación escribimos de la forma

y00 ��2 +

6

x

�y0 +

�6x+ 12

x2

�y = 0.

50

Como P (x) = ��2 +

6

x

�la segunda solución está dado por y2 = vy1.

Determinemos v(x):

v(x) =

Z1

[x3]2e

Z� 2+6

x

!dx

dx =

Z1

x6eR

2+6

x

!dx

dx =R 1

x6e2Rdx+6

R 1xdxdx

=

Z1

x6e2x+6lnjxjdx =

Z1

x6e2xelnjxj6dx =

Ze2x

x6x6dx =

Ze2xdx =

e2x

2.

Como y2 = vy1;entonces y2 =x3e2x

2,

o simplemente y2 = x3e2x.Por tanto la solución general es

y = C1x3 + C2x

3e2x.

ECUACIONES LINEALES HOMOGÉNEAS CON COEFICIENTESCONSTANTES

Sea la ecuación lineal homogénea de segundo orden

y00 + P (x)y0 +Q(x)y = 0.

Consideremos el caso en el que P (x) y Q(x), son constantes, es decir

y00 + py01 + qy = 0. (1)

donde los coe�cientes p y q son constantes reales. La ecuación (1) se llamaecuación lineal homogénea con coe�cientes constantes. Supongase que la funciónexponencial

y =emx, (2)

donde m es una constante, es una posible solución de la ecuación (1), cuyasderivadas son

y0 = memx

y00 = m2emx.

51

Sustituyendo (2) y sus derivadas en (1), tenemos

m2emx + pmemx + qemx = 0

(m2 + pm+ q)emx = 0.

Esta ecuación se satisface solamente cuando la ecuación caracteristica oecuación auxiliar sea cero. Es decir

m2 + pm+ q = 0.

Las raices m1 y m2 se obtienen mediante la fórmula general

m =�p�

pp2 � 4q2

.

Ahora debemos analizar, al igual como en álgebra los casos en el que eldiscriminante p2 � 4q, sea positiva, cero y negativo.

Raices Reales y Distintas.Si, p2�4q > 0 , entoncesm1 ym2 son raices reales y distintas, y las soluciones

son:

y1 =em1x, y2 =em2x.

Ademas y1 y y2 son linealmente independientes, puesto que:

y1y2=em1x

em2x=e(m1�m2)x 6= Ctte::

Por tanto la solución general de la ecuación (1) es

y = C1em1x + C2em2x.

Raices Reales e Iguales.Si p2�4q = 0, entoncesm1 ym2 son raices reales e iguales, es decirm1 = m2

y obtenemos solamente una sola solución: y1 =em1x con m1 = �P

2.

Sin embargo, podemos obtener la segunda solución �nalmente independienteutilizando el método de "reducción de orden"Sea: y1 =e(�

P2 )x, siendo

52

v =

Z1

y21�e�

RP (x)dxdx =Z

1he��

P2 xi2 �e�p R dxdx = Z 1

e�px�e�pxdx =

Zdx = x.

De manera que

y2 = vy1 =) y2 = xem1x.

Por tanto, la solución general de la ecuación (1) es

y = C1em1x + C2xem2x.

Raices Complejas.Si p2 � 4q < 0, entonces las raices m1 y m2 son raices complejas conjugadas

y escribimosm1 = �+ i� y m2 = �� i�; donde i =

p�1; � y � 2 R:

Con � = p2 y � =

p4q�p22 .

Luego las soluciones son:

y1 =e(�+i�)x, y2 =e(��i�)x.

Pero, estamos interesados solamente en soluciones de funciones reales, paraello consideremos la fórmula de Euler

ei� = cos � + i sen �, � 2 R.

Hacemos

em1x =e(�+i�)x =e�xei�x =e�x(cos�x+ i sen�x),

em2x =e(��i�)x =e�xe�i�x =e�x(cos�x� i sen�x),

donde:

cos(��x) = cos�x, sen(��x) = � sen�x.

53

De manera que:

y1 =em1x + em2x

2=e�x(cos�x+ i sen�x) + e�x(cos�x� i sen�x)

2

=e�x cos�x+ i e�x sen�x+ e�x cos�x� i e�x sen�x

2=2e�x cos�x

2=e�x cos�x.

y2 =em1x � em2x

2i=e�x(cos�x+ i sen�x)� e�x(cos�x� i sen�x)

2i=2i e�x sen�x

2i=e�x sen�x.

Por tanto la solución general de la ecuación (1) es

y = C1e�x cos�x+ C2e�x sen�x.

Ejemplo.

Hallar la solución general de la ecuación:

y00 + y0 � 6y = 0

Solución.Es una ecuación homogénea de coe�cientes constantes. La ecuación carac-

terística de la ecuación inicial es

m2 +m� 6 = 0

(m+ 3)(m� 2) = 0

m+ 3 = 0, m� 2 = 0

m1 = �3, m2 = 2.

Como las raices son reales y distintas, las soluciones son:

y1 =e�3x, y2 =e2x

Además y1 y y2 son linealmente independientes puesto que:

y1y2=e�3x

e2x=e�3x�2x =e�5x 6= Ctte:

Por tanto la solución general es:

54

y = C1e�3x + C2e2x.

Ejemplo.

Hallar la solución general de la siguiente ecuación:

y00 � 2y0 � y = 0

Solución.Es una ecuación homogénea de coe�cientes constantes. La ecuación carac-

terística es

m2 � 2m� 1 = 0

m =�(�2)�

q(�2)2 � 4(�1)2

=2�p8

2=2� 2

p2

2= 1�

p2.

Luego:m1 = 1�

p2,

m2 = 1�p2,

Como las raices son reales y distintas, de modo que las soluciones son

y1 =e(1+p2)x, y2 =e(1�

p2)x.

Vemos si son linealmente independientes y1 y y2

y1y2=e(1+

p2)x

e(1�p2)x

=e[(1+p2)x]�[(1�

p2)x] =e2

p2x 6= Ctte:

Por tanto, la solución general es

y = C1e(1+p2)x + C2e(1�

p2)x.

Ejemplo.

Hallar la solución general de la ecuación

55

y00 + 4y0 + 4y = 0.

Solución.La ecuación caracteristica es

m2 + 4m+ 4 = 0

(m+ 2)2 = 0

m+ 2 = 0

m = �2.

Siendo m1 = m2 = �2, raices reales e iguales, las soluciones son:

y1 =e�2x, y2 = xe�2x.

Por tanto, la solución general es

y = C1e�2x + C2xe�2x.

Ejemplo.Determinar la solución general de la ecuación

y00 + y0 + y = 0.

Solución.La ecuación característica de la ecuación dada es

m2 +m+ 1 = 0.

Aplicando la fórmula general, tenemos

m =�1�

p1� 4

2=�1�

p�3

2= �1

2� 12

p3p�1 = �1

2� 12

p3i,

Son raices complejas y conjugadas, es decir

m1 = �1

2+

p3i

2y m2 = �

1

2�p3i

2.

56

Donde � = �12y � =

p3

2, entonces las soluciones son

y1 =e�x cos�x =e�12x cos

p3

2x,

y2 =e�x sen�x =e�12x sen

p3

2x.

Por tanto, la solución general es:

y = C1e�x cos�x+ C2e�x sen�x

y = C1e�12x cos

p3

2x+ C2e�

12x sen

p3

2x.

MÉTODO DE LOS COEFICIENTES INDETERMINADOS

Consideremos la ecuación diferencial lineal no homogenea

y00 + P (x)y0 +Q(x)y = R(x). (1)

Estudiamos para aquellos casos que se conoce la solución general yg(x) dela correspondiente ecuación homogenea

y00 + P (x)y0 +Q(x)y = 0. (2)

Además, si yp(x) es cualquier solución particular de (1), entonces

y = yg(x) + yp(x),

Es la solución general de la ecuación (1). El método de los coe�cientesindeterminados es un procedimiento que nos permite hallar la solución particularyp, cuando la ecuación (1) tiene la forma

y00 + py0 + qy = R(x) (3)

donde p; q son constantes y R(x) es una función exponencial (eax), un poli-nomio(a0x

n + a1xn�1 + ::::::: + an�1x + an), una función trigonométrica como

(cosx�; sen�x); o una combinación lineal de tales funciones

57

R(x) = (a0xn + a1x

n�1 + :::::::+ an�1x+ an)eax

�cos�xsen�x

.

Si R(x) = lnx, R(x) = 1x , R(x) = tanx, :::, etc. el método no funciona.

Primero, analizaremos la ecuación (3) de la forma:

y00 + py0 + qy =eax. (4)

Supongase que la función exponencial

yp = Aeax, (5)

es una posible solución particular de la ecuación (4), donde A es el coe�cienteindeterminado que debe calcularse. Cuyas derivadas de (5) son:

y0p = Aaeax; y00p = Aa2eax.

Sustituyendo (5) y sus derivadas en (4), tenemos

Aa2eax + pAaeax + qAeax =eax

factorizando

A(a2 + pa+ q)eax =eax

A =1

a2 + pa+ q. (6)

Luego sustituyendo el valor de A en la ecuación (5), obtenemos la soluciónparticular, excepto cuando el denominador de (6) sea cero, esto ocurre cuandoa es una raíz de la ecuación característica:

m2 + pm+ q = 0.

Ahora intentamos de la forma

yp = Axeax. (7)

58

Siendo sus derivadas

y0p = Aeax +Aaxeax

y00p = Aaeax +Aaeax +Aa2xeax = 2Aaeax +Aa2xeax.

Sustituyendo (7) y sus derivadas en (4), resulta

2Aaeax +Aa2xeax + p(Aeax +Aaxeax) + qAxeax =eax.

2Aaeax +Aa2xeax + pAeax + pAaxeax + qAxeax =eax

A(a2 + pa+ q)xeax +A(2a+ p)eax =eax

Como a es una raiz de (4), entonces se reduce a

A(2a+ p)eax =eax

A(2a+ p) = 1

A =1

2a+ p.

Sustituyendo, el valor de A en la ecuación (7), obtenemos la solución partic-

ular, salvo cuando a = �p2. Finalmente intentamos de la forma

yp = Ax2eax (8)

Siendo

y0p = 2Axeax +Aax2eax

y00p = 2Aeax+2Aaxeax+2Aaxeax+Aa2x2eax = 2Aeax+4Aaxax+Aa2x2eax.

Sustituyendo (8) y sus derivadas en la ecuación (4), tendremos

A(a2 + pa+ q)x2eax + 2A(2a+ p)xeax +Aax2eax =eax

2Aeax =eax

de manera que

59

A =1

2.

Ahora analizamos la ecuacion (3) de la forma

y00 + py0 + qy = a0xn + a1x

n�1 + � � �+ an�1x+ an, (9)

es natural suponer que la solución particular también es un polinomiodegrado n

yp = A0xn +A1x

n�1 + � � �+An�1x+An; (10)

Sustituyendo (10) en (9) y sus derivadas e igualando los coe�cientes depotencias iguales de x obtenemos los coe�cientes indeterminados A0; A1; : : : ; An:El procedimiento falla cuando q = 0, entonces intentamos

yp = x(A0xn +A1x

n�1 + : : :+An�1x+An).

En el caso particular si p = 0 y q = 0, la solución obtenemos por integracióndirecta.Finalmente, analizaremos cuando la ecuación (3) es de la forma

y00 + py0 + qy = sen�x (11)

Supongase que, la solución particular de la ecuación (11) es

yp = A cos�x+B sen�x (12)

Sustituyendo (12) y sus derivadas en (11) e igualando los coe�cientes decos�x y sen�x obtenemos los coe�cientes indeterminados A y B. El proced-imiento falla cuando cos�x; o sen�x es una solución de la ecuación (10), o bieni� es una raiz de la ecuación caracteristica. Entonces intentamos de la forma:

yp = x(A cos�x+B sen�x)

= Ax cos�x+Bx sen�x.

Ejemplo.

Hallar la solución general de la ecuación

60

y00 � 2y0 � 3y = 4e�3x

Solución.Primero determinamos la solución general de la ecucación homogénea.

y00 � 2y0 � 3y = 0:

La ecuación característica es

m2 � 2m� 3 = 0(m� 3)(m+ 1) = 0.

Siendo las raices

m1 = 3; m2 = �1.

De modo que la solución general es

yg = C1e3x + C2e�x.

Ahora determinamos la solución particular yp. Como a = �3 es diferente dem1 y m2 entonces planteamos de la forma

yp = Ae�3x

y0p = �3Ae�3x

y00p = 9Ae�3x

Sustituyendo, estos valores en la ecuaciones no homogeneas dada, tenemos

9Ae�3x � 2(�3Ae�3x)� 3Ae�3x = 4e�3x

9Ae�3x + 6Ae�3x � 3Ae�3x = 4e�3x

12Ae�3x = 4e�3x

A =1

3

Luego, la solución particular es

yp =1

3e�3x.

61

Por tanto la solución general es

y = yg + yp = C1e3x + C2e�x +1

3e�3x.

Ejemplo.

Hallar la solución general de la ecuación

y00 + 2y0 + y = 3e�x. (1)

Solución.Primero determinamos la solución general yg de la ecuación homogénea

y00 + 2y0 + y = 0 (2)

La ecuación característica es

m2 + 2m+ 1 = 0

(m+ 1)2 = 0

son raices reales y repetidas

m1 = m2 = �1

En consecuencia la solución general de la ecuación (2) es

yg = C1e�x + C2xe�x.

Ahora determinamos la solucion particular yp de la ecuacion (1). Comoa = �1 es una raíz repetida de la ecuación característica, entonces buscamos lasolución particular de la forma

yp = Ax2e�x.

Las derivadas son

y0p = 2Axe�x �Ax2e�x

y00p = 2Ae�x � 2Axe�x � 2Axe�x +Ax2e�x

y00p = 2Ae�x � 4Axe�x +Ax2e�x

62

Sustituyendo yp y sus derivadas en la ecuación (1), se tiene

2Ae�x � 4Axe�x +Ax2e�x + 4Axe�x � 2Ax2e�x +Ax2e�x = 3e�x

2Ae�x = 3e�x.

Igualamos los coe�cientes, obtenemos.

2A = 3

A =3

2.

En consecuencia la solución particular de la ecuación (1) es

yp = Ax2e�x

yp =3

2x2e�x.

Por tanto la solución general de (1) es:

y = yg + yp

y = C1e�x + C2xe�x +3

2x2e�x

Ejemplo.

Hallar la solución general de la ecuación

y00 � 3y0 + 2y = (x+ 1)2. (1)

Solución.

Primero determinemos la solución general yg de la ecuación homogénea

y00 � 3y0 + 2y = 0 (2)

Siendo la ecuación característica

m2 � 3m+ 2 = 0

(m� 2)(m� 1) = 0,

63

siendo las raices

m1 = 1; m2 = 2

En consecuencia, la solución general de la ecuación (2) es

yg = C1ex + C2e2x

Ahora determinemos la solución particular yp de la ecuación no homogénea(1). Buscamos de la forma:

yp = Ax2 +Bx+ C,

siendo sus derivadas

y0p = 2Ax+B

y00p = 2A.

Sutituyendo yp y sus derivadas en la ecuación (1), se tiene:

2A� 3(2Ax+B) + 2(Ax2 +Bx+ C) = x2 + 2x+ 1

2A� 6Ax� 3B + 2Ax2 + 2Bx+ 2C = x2 + 2x+ 1

2Ax2 + (�6A+ 2B)x+ (2A� 3B + 2C) = x2 + 2x+ 1

Igualamos los coe�cientes de potencias iguales de x, obtenemos el sistema8<: 2A = 1;�6A+ 2B = 2;

2A� 3B + 2C = 1:

Donde

A =1

2; B =

5

2; C =

15

4.

Luego, la solución particular de la ecuación (1) es

yp =1

2x2 +

5

2x+

15

4.

64

Por tanto, la solución general de la ecuación (1) es

y = C1ex + C2e2x +1

2x2 +

5

2x+

15

4.

Ejemplo.Hallar la solución general de la ecuación

y00 + y = cosx+ senx. (1)

Solución.Comenzamos determinando la solución general yg de su correspondiente

ecuación homogenea

y00 + y = 0 (2)

La ecuación caracteristica es

m2 + 1 = 0,

sus raices son complejas y conjugadas

m1 = i; m2 = �i

De manera que la solución gerneral de la ecuación (2) es

yg = C1 cosx+ C2 senx

Seguidamente determinamos la solución particular yp de la ecuación (1).Como R(x) = cosx+senx es una solución de la ecuación homogénea. Buscamosla solución particular de forma

yp = x(A cosx+B senx)

yp = Ax cosx+Bx senx

Sus derivadas son

65

y0p = A cosx�Ax senx+Bx senx+Bx cosx

y0p = (Bx+A) cosx+ (�Ax+B) senx

y00p = B cosx� (Bx+A) senx�A senx+ (�Ax+B) cosx

y00p = (�Ax+ 2B) cosx� (Bx+ 2A) senx

Sustituyendo yp y sus derivadas, tenemos:

(�Ax+ 2B) cosx� (Bx+ 2A) senx+Ax cosx+Bx senx = cosx+ senx

�Ax cosx+2B cosx�Bx senx� 2A senx+Ax cosx+Bx senx = cosx+senx

2B cosx� 2A senx = cosx+ senx

Igualamos los coe�cientes de cosx y senx; obtenemos��2A = 1;2B = 1:

Donde

A = �12, B =

1

2.

Reemplazando estos coe�cientes en la solución particular es

yp = �1

2x cosx+

1

2x senx

Por tanto, la solución general de (1) es:

y = yg + yp = C1 cosx+ C2 senx�1

2x cosx+

1

2x senx.

Ejemplo.

Hallar la solución general de la ecuación

y00 � y0 � 6y = 39 sen 3x. (1)

66

Solución.Sea la ecuación homogénea

y00 � y0 � 6y = 0. (2)

La ecuación característica es

m2 �m� 6 = 0

(m� 3)(m+ 2) = 0

Las raices son

m1 = 3; m2 = �2

En consecuencia la solución general de la ecuación (2) es

yg = C1e3x + C2e�2x

Planteamos la solución particular de la forma

yp = A cos 3x+B sen 3x.

Y sus derivadas son

y0p = �3A sen 3x+ 3B cos 3x

y00p = �9A cos 3x� 9B sen 3x.

Sustituyendo la solución particular y sus derivadas en (1), tendremos

�9A cos 3x� 9B sen 3x� (�3A sen 3x+ 3B cos 3x)� 6(A cos 3x+B sen 3x) =

39 sen 3x�9A cos 3x�9B sen 3x+3A sen 3x�3B cos 3x�6A cos 3x�6B sen 3x = 39 sen 3x

�15A cos 3x� 15B sen 3x+ 3A sen 3x� 3B cos 3x = 39 sen 3x

(�15A� 3B) cos 3x+ (3A� 15B) sen 3x = 39 sen 3x.

67

Igualando coe�cientes de cos 3x y sen 3x, obtenemos el sistema��15A� 3B = 0;3A� 15B = 39:

=) �3B = 15A =) B = � 153 A = �5A:

=) 3A� 15(�5A) = 39, =) 3A+ 75A = 39, =) A = 3978 =

12 . .=) B = � 52 .

De manera que

A =1

2

B = �52.

Reemplazando los coe�cientes en yp, La solución particular es:

yp =1

2cos 3x� 5

2sen 3x

Por tanto la solución general de (1) es:

y = yg + yp = C1e3x + C2e�2x +1

2cos 3x� 5

2sen 3x:

VARIACIÓN DE PARÁMETROS

Ahora desarrollaremos un método más general, llamado "variación de parámet-ros", para determinar una solución particular de la ecuación

y00 + P (x) y0 +Q (x) y = R (x). (1)

Para emplear este método, es necesario conocer un conjunto fundamental desoluciones de la ecuación homogénea

y00 + P (x) y0 +Q (x) y = 0, (2)

donde P (x), Q(x) y R(X) son funciones continuas en un intervalo.Supongase que y1 y y2 son dos soluciones L.I. de la ecuación (2), entonces

la solución general de la ecuación homogénea (2) es

68

yg = C1y1 (x) + C2y2 (x). (3)

El método es análogo a la "reducción de orden", es decir, sustituimos lasconstantes C1 y C2 por las funciones desconocidas v1 (x), v2 (x) e intentamoshallar v1 y v2 tal que

yp = v1y1 + v2y2, (4)

sea la solución particular de la ecuación (1). Para este propósito debemoscalcular las derivadas

y0p = v1y01 + v

01y1 + v2y

02 + v

02y2

y0p = (v1y01 + v2y

02) + (v

01y1 + v

02y2). (5)

Una segunda derivada complicaria más, entonces hacemos que el segundotérmino entre parentesis se anule

v01y1 + v02y2 = 0. (6)

La ecuación (5) se reduce a

y0p = v1y01 + v2y

02. (7)

Derivando una vez más, se tiene

y00p = v1y001 + v

01y01 + v2y

002 + v

02y02. (8)

Sustituyendo las ecuaciones (4), (7) y (8) en (1), obtenemos

(v1y001 + v

01y01 + v2y

002 + v

02y02) + P (v1y

01 + v2y

02) +Q (v1y1 + v2y2) = R (x).

Reagrupando los términos, resulta

v1 (y001 + Py

01 +Qy1) + v2 (y

002 + Py

02 +Qy2) + v

01y01 + v

02y02 = R (x),

como y1, y2 son soluciones de la ecuación (2), los dos términos entre paran-tesis se anulan, para tener

69

v01y01 + v

02y02 = R (x). (9)

Las ecuaciones (6) y (9) constituyen un sistema lineal de dos ecuaciones conlas incognitas v01 y v02.

v01y1 + v02y2 = 0

v01y01 + v

02y02 = R (x)

Resolviendo el sistema por la regla de Cramer, tenemos

v01 =�y2R (x)W (y1; y2)

; v02 =y1R (x)

W (y1; y2). (10)

Donde

W (y1; y2) =

���� y1 y2y01 y02

����,es el Wronskiano de y1 y y2, con W (y1; y2) 6= 0.Integrando las ecuaciones de (10), tendremos

v1 =

Z �y2R (x)W (y1; y2)

dx, v2 =

Zy1R (x)

W (y1; y2)dx.

Finalmente la solución particular de la ecuación (1) es

yp = y1

Z �y2R (x)W (y1; y2)

dx+ y2

Zy1R (x)

W (y1; y2)dx.

Por tanto la solución general de la ecuación (1) es

y = yg + yp.

Ejemplo.

Determinar la solución general de

4y00 + 4y0 + y = 3ex. (11)

Solución.

La ecuación (11) escribimos en la forma (1)´:

y00 + y0 +1

4y =

3

4ex. (12)

70

primero, determinemos la solución general yg, de la ecuación homogénea

y00 + y0 +1

4y = 0. (13)

Siendo la ecuación caracteristica

m2 +m+1

4= 0

m =�p�

pp2 � 4q2

=�1�

p1� 1

2= �1

2.

Luegom1 = m2 = � 12 .

Como las raices son reales y repetidas, las soluciones son

y1 =e�12x, y2 = xe�

12x.

Así, la solución general de la ecuación (13) es

yg = C1e�12x + C2xe�

12x.

Ahora calculemos el Wronskiano de y1, y2.

W (y1; y2) =

���� y1 y2y01 y02

���� =����� e�

12x xe�

12x

�12e�

12x e�

12x � 1

2xe�

12x

����� =e�x � 12xe�x +1

2xe�x =e�x.

De manera que,

v1 =

Z �y2R (x)W (y1; y2)

dx =

Z �xe� 12x � 3

4ex

e�xdx =

Z �34xe

12x

e�xdx = �3

4

Zxe 3

2xdx

Integrando por partes, tenemosu = x =) du = dx

v =

Ze 3

2xdx =2

3e 3

2x.

71

v1 = �3

4

Zxe 3

2xdx = �34

�2

3xe

32x � 2

3

Ze32xdx

�= �1

2xe 3

2x +1

3e 3

2x.

Para v2, se tiene

v2

Zy1R (x)

W (y1; y2)dx =

Z e�12x3

4ex

e�xdx =

3

4

Ze 3

2x =1

2e 3

2x.

Luego, la solución particular es

yp =

��12xe

32x +

1

3e32x

�e� 1

2x +1

2e 3

2x � xe� 12x = �1

2xex + 1

3ex + 1

2xex =

1

3ex.Por tanto, la solución general de la ecuación (11) es

y = yg + yp

y = C1e�12x + C2xe�

12x +

1

3ex.

Ejemplo.Hallar la solución particular de la ecuación

y00 + 4y = tan 2x (14)

Solución.

Determinemos la solución general de la ecuación homogenea

y00 + 4y = 0. (15)

La ecuanción caracteristica es

m2 + 4 = 0

m =p�4 = �2i,

son raices complejas y conjugadas, donde � = 0 y � = 2. De manera que la

solución general de la ecuación homogénea (15) es

72

yg = C1 cos 2x+ C2 sen 2x

Sea

y1 = cos 2x y y2 = sen 2x:

Ahora, calculamos el wronskiano de y1 y y2

W (y1; y2) =

���� y1 y2y01 y02

���� = ���� cos 2x sen 2x�2 sen 2x 2 cos 2x

���� = 2 cos2 2x+ 2 sen2 2x =2�cos2 2x+ sen2 2x

�= 2.

De manera que:

v1 =

Z �y2R (x)W (y1; y2)

dx =

Z � sen 2x � tan 2x2

dx = �12

Zsen 2x

sen 2x

cos 2xdx =

�12

Zsen2 2x

cos 2xdx

= �12

Z1� cos2 2xcos 2x

dx = �12

Z �1

cos 2x� cos 2x

�dx =

1

2

Z(cos 2x� sec 2x) dx

=1

2

�sen 2x

2� ln jsec 2x+ tan 2xj

2

�=1

4sen 2x� 1

4ln jsec 2x+ tan 2xj.

Para v2, tenemos

v2 =

Zy1R (x)

W (y1; y2)dx =

Zcos 2x tan 2x

2dx =

1

2

Zcos 2x

sen 2x

cos 2xdx =

1

2

Zsen 2xdx = �1

4cos 2x.

Luego, la solución particular es

73

yp =

�1

4sen 2x� 1

4ln jsec 2x+ tan 2xj

�cos 2x+

��14cos 2x

�sen 2x

=1

4sen 2x cos 2x� 1

4cos 2x ln jsec 2x+ tan 2xj � 1

4sen 2x cos 2x

yp = �1

4cos 2x ln jsec 2x+ tan 2xj.

TEMA No 4

SOLUCION EN SERIE DE POTENCIAS DEECUACIONES DIFERENCIALES

REPASO DE SERIE DE POTENCIAS

1. Una serie in�nita de la forma1Xn=0

anxn = a0 + a1x+ a2x

2 + a3x3 + � � � (1)

se llama una serie de potencias en x:

2. La serie.1Xn=0

an (x� x0)n = a0 + a1 (x� x0) + a2 (x� x0)2 + � � � (2)

es una serie de potencias en x � x0, que es algo más general que (1). Sinembargo (2) se puede reducir a (1) sustituyendo x� x0 por x:

3. Con relación a la disposición de sus puntos de convergencia, todas lasseries de potencias caen en algunas de las tres categorias siguientes

1).1Xn=0

n!xn = 1 + x+ 2!x2 + � � �

2).1Xn=0

xn

n!= 1 + x+

x2

2!+ � � �

74

3).1Xn=0

xn = 1 + x+ x2 + � � �

La primera serie es divergente para todo x 6= 0, la segunda es convergentepara todo x, la tercera es convergente para jxj < 1 y diverge para jxj > 1:

Radio de Convergencia.Toda serie de potencia en x, tiene un radio de convergencia R, que tiene

la propiedad de que, la serie converge sí jxj < 1 y diverge sí jxj > 1, donde0 6 R 61:

Para poder determinar el valor de R, consideremos. Sea una serie de con-stantes no nula

1Xn=0

un = u0 + u1 + u2 + � � �

por cálculo elemental, sí el límite

limn!0

����un+1un

���� = L,

existe, entonces por el "criterio del cociente" se asegura que:

Sí: L < 1, la serie converge.

Sí: L > 1, la serie diverge.

Para el caso de serie de potencias

1Xn=0

anxn = a0 + a1x+ a2x

2 + � � �

donde

limn!1

����an+1xn+1anxn

���� = jxj limn!1

����an+1an

���� = L.Para la serie de potencias

75

1Xn=0

an (x� x0)n = a0 + a1 (x� x0) + a2 (x� x0)2 + � � �

donde

limn!1

�����an+1 (x� x0)n+1an (x� x0)n

����� = jx� x0j limn!1

����an+1an

���� = L;

sí el límite existe, entonces.

Sí L < 1, la serie de potencias converge.

Sí L > 1, la serie de potencias diverge.

Sí L = 0, la prueba no es concluyente.

En concecuencia, la fórmula de radio de convergencia es

R = 1

limn!0

�����an+1an

�����= lim

n!0

���� anan+1����

sí el limite existe. Como 0 6 L 61 entonces R =1

L, si R = 0 si L =1 y

R =1 si L = 0.

Sí R es �nito y no nulo determina un "intervalo de convergencia" x0 �R 6

x 6 R + x0, tal que la serie es convergente en su interior y divergente en elexterior. Gra�camente tenemos

Una serie de potencias puede o no ser convergente en los puntos terminalesde su intervalo de convergencia.

Ejemplo.Determinar radio e intervalo de convergencia de la serie de potencias.

1Xn=1

(x+ 2)n

3nn.

76

Solución.

Primero determinamos radio de convergencia

R = limn!1

��������1

3nn1

3n+1 (n+ 1)

�������� = limn!1

����3n+1 (n+ 1)3nn

���� = limn!1

����3n3 (n+ 1)3nn

���� =

limn!1

����3 (n+ 1)n

���� =?Aplicando algunos arti�cios matemáticos, para levantar la indeterminada,

tenemos

R = limn!1

�������3 (n+ 1)

nn

n

������� = limn!1

��������3

�1 +

1

n

�1

�������� = 3 limn!1

��1 + 1n

�� = 3 � 1 = 3.

NOTA:1

1 = 0.

Siendo el radio de convergencia R = 3. Ahora determinemos el intervalo deconvergencia, utilizando el criterio del cociente

limn!1

����an+1an

���� = limn!1

���������(x+ 2)

n+1

3n+1 (n+ 1)

(x+ 2)n

3nn

��������� = limn!1

����� 3nn (x+ 2)n+1

3n+1 (n+ 1) (x+ 2)n

����� =limn!1

����n (x+ 2)3 (n+ 1)

���� = jx+ 2j limn!1

���� n

3 (n+ 1)

����= jx+ 2j lim

n!1

�������n

n3 (n+ 1)

n

������� = jx+ 2j limn!1

��������1

3

�1 +

1

n

��������� = jx+ 2j �

1

3< 1.

jx+ 2j < 3

�3 < x+ 2 < 3.

De modo que la serie de potencias converge para �5 < x < 1. Ahora

analizaremos los puntos terminales del intervalo I de convergencia.

Para x = �5, la serie de potencias se convierte en la serie canónica alternada

77

1Xn=1

(�5 + 2)n

3nn=

1Xn=1

(�3)n

3nn=

1Xn=1

(�1)n

n,

la cual es convergente.

Para x = 1, la serie se convierte en la serie armonica.

1Xn=1

(1 + 2)n

3nn=

1Xn=1

1

n,

la cual es divergente. Por tanto, el intervalo de convergencia es

�5 6 x < 1.

Ejemplo.Determinar radio e intervalo de convergencia de la serie

1Xn=1

4

n2 + 2(x� 3)n

Solución.Primero determinamos radio de convergencia

R = limn!1

���� anan+1���� = lim

n!1

��������4

n2 + 24

(n+ 1)2+ 2

�������� = limn!1

����� (n+ 1)2 + 2n2 + 2

����� =

limn!1

����n2 + 2n+ 3n2 + 2

���� = limn!1

�������n2 + 2n+ 3

n2

n2 + 2

n2

������� = limn!1

�������1 +

2

n+3

n2

1 +2

n2

������� = 1.

Siendo el radio de convergencia R = 1. Ahora determinamos el intervalo deconvergencia, aplicando el criterio del cociente.

limn!1

����an+1an

���� = limn!1

��������4

(n+ 1)2+ 2

(x� 3)n+1

4

n2 + 2(x� 3)n

�������� =limn!1

������ 4 (x� 3)n+1�n2 + 2

�h(n+ 1)

2+ 2i4 (x� 3)n

������ = limn!1

������ 4 (x� 3)n+1�n2 + 2

�h(n+ 1)

2+ 2i4 (x� 3)n

������78

= limn!1

����� (x� 3)�n2 + 2

�n2 + 2n+ 3

����� = jx� 3j limn!1

���� n2 + 2

n2 + 2n+ 3

���� =jx� 3j lim

n!1

�������n2 + 2

n2

n2 + 2n+ 3

n2

������� = jx� 3j limn!1

�������1 +

2

n2

1 +2

n+3

n2

�������= jx� 3j

�1

1

�= jx� 3j � 1 < 1.

De modo que la serie de potencias "Converge", como jx� 3j < 1 es decir

�1 < x� 3 < 1

�1 + 3 < x� 3 + 3 < 1 + 3

2 < x < 4,

siendo el intervalo de convergencia: 2 < x < 4.

Analizaremos los puntos terminales del intervalo de convergencia.

Para x = 2, se tiene1Xn=1

4

n2 + 2(x� 3)n =

1Xn=1

4

n2 + 2(2� 3)n =

1Xn=1

4 (�1)n

n2 + 2=

1Xn=1

4 (�1)n

n2 + 2

La serie es convergente.

Para x = 4, se tiene

1Xn=1

4

n2 + 2(x� 3)n =

1Xn=1

4

n2 + 2(4� 3)n =

1Xn=1

4 (1)n

n2 + 2ó

1Xn=1

4

n2 + 2

La serie es divergente. Por tanto, el intervalo de convergencia es: 2 6 x < 4.4. Sea f(x) una función continua y que admite derivadas de todo orden

en jxj < R con R > 0 de f (x) se puede representar mediante una serie depotencias de la forma

f (x) =1Xn=0

f (n) (0)

n!xn = f (0) + f 0 (0)x+

f 00 (0)

2!x2 + � � �

donde

an =f (n) (0)

n!.

79

Una forma de veri�car la validez de este desarrollo es mediante la fórmulade Taylor

f (x)1

=Pk=0

f (k) (0)

k!xn +Rn (x).

Donde Rn (x) es el resto

Rn (x) =f (n+1) (�)

(n+ 1)!,

para � 2 (0; 1).

Ejemplo.Sean los dsarrollos alrededor del punto x0 , llamadas series Maclaurin

ex =

1Xn=0

xn

n!= 1 + x+

x2

2!+x3

3!+ � � �

senx =1Xk=0

(�1)n x2n+1

(2n+ 1)!= x� x

3

3!+x5

5!� x

7

7!+ � � �

cosx =1Xn=0

(�1)n x2n

(2n!)= 1� x

2

2!+x4

4!� x

6

6!+ � � �

5. Una función que admite un desarrolla en serie de potencias de la forma

f (x) =

1Xn=0

an (x� x0)n,

en algún entorno del punto x0, se dice que f (x) es "analitica" en x0.

Ejemplo.Las funciones ex, senx y cosx son analíticas en x0 = 0.SOLUCIÓN POR SERIE DE POTENCIAS DE ECUACIONES

DIFERENCIALES DE PRIMER ORDEN

Ejemplo.Consideremos la ecuación diferencial

y0 + y = 0 (1)

Solución.Suponemos que la solución es en forma de serie de potencias

80

y = a0 + a1x+ a2x2 + a3x

3 + � � �+ anx+ � � � (2)

que convenge para jxj < R, con R > 0 y que tiene una solución análitica enel origen, es decir, x0 = 0:

La ecuación (2) puede derivarse término a término.

y0 = a1 + 2a2x+ 3a3x2 + 4a4x

3 + � � �+ (n� 1)

an�1xn�2 + nanx

n�1 + (n+ 1)an+1xn + � � � (3)

Sustituyendo (2) y (3) en (1), tenemos(a1+2a2x+3a3x

2+� � �+(n� 1) an�1xn�2+nanxn�1+(n+1)an+1xn+� � � )+

+�a0 + a1x+ a2x

2 + a3x3 + � � �+ an�1xn�1 + anxn + an+1xn+1 + � � �

�= 0.

Agrupando los coe�cientes de potencia iguales de x, tenemos

(a1 + a0) + (2a2 + a1)x+ (3a3 + a2)x2 + (4a4 + a3)x

3 + � � �+ (nan +

an�1)xn�1 + [(n+ 1) an+1 + an]x

n + :::: = 0.

Igualando a cero los coe�ciente, resulta.a1 + a0 = 0) a1 = �a0

2a2 + a1 = 0) a2 = �a12=

a01 � 2

3a3 + a2 = 0) a3 = �a23= � a0

1 � 2 � 3

4a4 + a3 = 0) a4 = �a34=

a01 � 2 � 3 � 4

...nan + an�1 = 0 entonces an+1 = (�1)n

a01 � 2 � 3 � � � � � (n+ 1) .

Sustituyendo estos coe�cientes en la ecuación (2) obtenemos la solución de(1) , en serie de potencias, esto es

y = a0 � a0x+a02!x2 � a0

3!x3 +

a04!x4 + � � �+ a0

n!xn � a0

(n+ 1)!xn+1 + � � �

y = a0

�1� x+ x

2

2!� x

3

3!+x4

4!+ � � �+ x

n

n!� xn+1

(n+ 1)!+ � � �

81

ó bieny =

a0e�x.Ejemplo.

Sea la ecuación diferencial

(1 + x) y0 + y = 0 (1)

Determinar una solución en serie de potencias en el entorno x0 = 0.Solución.

Suponemos que la solución es de la forma

y = a0 + a1x+ a2x2 + a3x

3 + a4x4 + � � � (2)

Siendo las derivadas

y0 =

a1+2a2x+3a3x2+4a4x

3+� � �+(n�1)an�1xn�2+nanxn�1+(n+1)an+1xn+� � �(3)

Sustituyendo los desarrollos y y y0 en (1), tenemos

(1 + x)�a1 + 2a2x+ 3a3x

2 + 4a4x3 + � � �

�+�

a0 + a1x+ a2x2 + a3x

3 + a4x4 + � � �

�= 0

(a1 + 2a2x+ 3a3x2 + 4a4x

3 + � � �+ a1x+ 2a2x2 + 3a3x3 + 4a4x4 + � � � ) + (a0 +

a1x+ a2x2 + a3x

3 + a4x4 + � � � ) = 0.

Agrupamos los términos semejantes

(a1 + a0) + (2a2 + 2a1)x+ (3a3 + 3a2)x2 + (4a4 + 4a3)x

3 + � � � = 0.

82

Luego igualamos los coe�cientesa1 + a0 = 0) a1 = �a0

2a2 + 2a1 = 0) a2 = �a1 = a0

3a3 + 3a2 = 0) a3 = �a2 = �a0

4a4 + 4a3 = 0) a4 = �a3 = a0

...

Sustituyendo estos coe�cientes en (2) tenemos la solución

y = a0�1� x+ x2 � x3 + x4 + � � �

ó simplemente

y = a0

�1

1 + x

�.

Ejemplo.

Resolver la ecuación

2(x� 1)y0 � 3y = 0, (1)

por serie de potencias entorno al punto x0 = 0.Solucion.

Suponemos que la solucion es de la forma

y = a0 + a1x+ a2x2 + a3x

3 + a4x4 + � � � (2)

Su derivada es

y0 = a1 + 2a2x+ 3a3x2 + 4a4x

3 + � � �+ (n� 1)an�1xn�2 + nanxn�1 + (n+

1)an+1xn + � � � (3)

Sustituyendo (2) y (3) en (1), tenemos

83

2 (x� 1)�a1 + 2a2x+ 3a3x

2 + 4a4x3 + � � �

��

3�a0 + a1x+ a2x

2 + a3x3 + a4x

4 + � � ��= 0�

2a1x+ 4a2x2 + 6a3x

3 + 8a4x4 + � � � � 2a1 � 4a2x� 6a3x2 � 8a4x3 � � � �

�+�

�3a0 � 3a1x� 3a2x2 � 3a3x3 � 3a4x4 � � � ��= 0

Agrupamos los términos semejantes

(�2a1 � 3a0) + (�a1 � 4a2)x+ (a2 � 6a3)x2 + (3a3 � 8a4)x3 +

(5a4 � 10a5)x4 + � � � = 0.

Igualando términos se tiene;

�2a1 � 3a0 = 0) a1 = �3a02

�a1 � 4a2 = 0) a2 = �a14=3a08

a2 � 6a3 = 0) a3 =a26=3a048

3a3 � 8a4 = 0) a4 =3a38=3a0128

...Siendo la solución

y = a0

�1� 3

2x+

3

8x2 +

3

48x3 +

3

128x4 + � � �

y = a0

�1

2 (x� 1)

�= a0

�1

2x� 2

�.

ECUACIONES DIFERENCIALES DE SEGUNDO ORDEN

Puntos Ordinarios.Consideramos la ecuación lineal homogénea de segundo orden

y00 + P(x)y0 +Q(x)y = 0 (1)

84

donde P (x) y Q (x) son funciones continuas de x para nuestro análisis nosrestringiremos al caso en que P (x) y Q (x) deben ser análiticas en x0, lo quesigni�ca que P (x) y Q (x) tiene un desarrollo en serie de potencias, válida enalgún entorno del punto x0, en tal caso se dira que x0 es "punto ordinario" dela ecuación (1), en caso contrario es un "punto singular".Ejemplo.Resolver la ecuación

y00 + y = 0, (2)

en serie de potencias de x.Solución.Las funciones P (x) = 0, Q (x) = 1, ambas a la vez son analíticas en el punto

x0 = 0 (más en general para todo x), de manera que x0 = 0 es un puntoordinario.Buscamos la solución de la forma

y = a0 + a1x+ a2x2 + a3x

3 + ::::+ an�1xn�1 + anx

n + an+1xn+1 + ::: (3)

cuyas derivadas son

y0 = a1 + 2a2x+ 3a3x2 + 4a4x

3 + � � �+ (n� 1) an�1xn�2 + nanxn�1 + (n+

1)an+1xn + (n+ 2)an+2x

n+1 + � � �y00 = 2a2 + 6a3x+ 12a4x

2 + � � �+ (n� 2) (n� 1) an�1xn�3 +

(n� 1)nanxn�2 + n (n+ 1) an+1xn�1 + (n+ 1) (n+ 2) an+2xn + � � �(4)

Sustituyendo los desarrollos (3) y (4) en (2), tenemos[2a2 + 6a3x+ 12a4x

2 + � � �+ (n� 2) (n� 1) an�1xn�3 + (n� 1)nanxn�2 +

n (n+ 1) an+1xn�1 + (n+ 1) (n+ 2) an+2x

n + � � � ]+

+�a0 + a1x+ a2x

2 + a3x3 + a4x

4 + � � ��= 0

Agrupamos los términos semejantes:

(2a2 + a0) + (6a3 + a1)x+ (12a4 + a2)x2 + � � �+

[(n� 2) (n� 1) an�1 + an�3]xn�3 + [(n� 1)nan + an�2]xn�2+ [n (n+ 1) an+1 + an�1]x

n�1 + [(n+ 1) (n+ 2) an+2 + an]xn + � � � = 0

85

Igualando a cero cada coe�ciente, tenemos2a2 + a0 = 0) a2 = �

a01 � 2

6a3 + a1 = 0 ) a3 = �a1

1 � 2 � 3

12a4 + a2 = 0) a4 = �a23 � 4 =

a01 � 2 � 3 � 4

20a5 + a3 = 0) a5 = �a34 � 5 =

a11 � 2 � 3 � 4 � 5

Sustituyendo estos coe�cientes en (3), obtenemos la solución.

y = a0 + a1x�a02!x2 � a1

3!x3 +

a04!x4 +

a15!x5 � � � �

y = a0

�1� x

2

2!+x4

4!� x

6

6!+ � � �

�+ a1

�x� x

3

3!+x5

5!� x

7

7!+ � � �

�,

o simplemente

y = a0 cosx+ a1 senx

Ejemplo.

Resolver la ecuación

(1 + x) y00 + y0 = 0 (6)

en serie de potencias.

Solución.

Escribimos la ecuación en forma canónica

y00 +1

1 + xy0 = 0.

Como P (x) =1

1 + xy Q(x) = 0 son ambas son análiticas en el origen

x0 = 0, entonces x0 = 0 es un punto ordinario.Suponemos que la solución es de forma siguiente

y = a0 + a1x+ a2x2 + a3x

3 + a4x4 + � � � (7)

o bien

86

y =1Xn=0

anxn

Las derivadas son

y0 =1Xn=0

nanxn�1

y00 =1Xn=0

(n� 1)nanxn�2

Sea n n+ 1 para y0; entonces:

y0 =

1Xn=0

(n+ 1) an+1xn

Sea n n+ 2 para y00;entonces

y00 =1Xn=0

(n+ 1) (n+ 2) an+2xn

Sustituyendo los desarrollos y0 y y00 en (6) , tenemos

(1 + x)1Xn=0

(n+ 1) (n+ 2) an+2xn +

1Xn=0

(n+ 1) an+1xn = 0

1Xn=0

(n+ 1) (n+ 2) an+2xn +

1Xn=0

(n+ 1) (n+ 2) an+2xn+1 +

1Xn=0

(n+ 1) an+1xn = 0

Para la segunda sumatoria sea: n n� 11Xn=0

(n+ 1) (n+ 2) an+2xn +

1Xn=1

n (n+ 1) an+1xn +

1Xn=0

(n+ 1) an+1xn = 0

1 � 2a2 +1Xn=1

(n+ 1) (n+ 2) an+2xn +

1Xn=1

n (n+ 1) an+1xn + a1 +

1Xn=1

(n+ 1) an+1xn = 0

87

Agrupamos los términos semejantes, se tiene

(1 � 2a2 + a1)+1Xn=1

[(n+ 1) (n+ 2) an+2 + n (n+ 1) an+1 + (n+ 1) an+1]xn = 0

(1 � 2a2 + a1) +1Xn=1

h(n+ 1) (n+ 2) an+2 + n (n+ 1)

2an+1

ixn = 0

Igualamos a cero los coe�cientes,resulta1 � 2a2 + a1 = 0) a2 = �

a11 � 2

(n+ 1) (n+ 2) an+2 + n (n+ 1)2an+1 = 0

siendo

an+2 = �n (n+ 1)

2an+1

(n+ 1) (n+ 2)= �n (n+ 1) an+1

(n+ 2)= �n (n+ 1)

(n+ 2)an+1, para

n : 1; 2; 3; : : :

Está ecuación es conocida como la "formula de recurrencia".a2 = �

a11 � 2

a3 = �2

3a2 =

��23

��� a11 � 2

�=a13

a4 = �3

2a3 =

��32

���a13

�= �a1

2

a5 = �12

5a4 =

��125

���a12

�=6

5a1

a6 = �10

3a5 = �

10

3

6

5a1 = �4a1

...

Sustituyendo estos coe�cientes , en la ecuación (7) ,obtenemos la solución

y = a0 + a1x�a12x2 +

a13x3 � a1

2x4 +

6

5x5 � 4a1x6 + � � �

88

Ejemplo.

Resolver por serie de potencias, la ecuación

y00 � (x+ 1) y = 0 (8)

Solución.Como P (x) = 0 yQ (x) = � (x+ 1) como las funciones, ambas son análiticas

en x0 = 0. Suponemos que la ecuación tiene la solución de la forma

y = a0 + a1x+ a2x2 + a3x

3 + a4x4 + � � � (9)

o bien

y =1Xn=0

anxn, con y0 =

1Xn=0

(n+ 1) an+1xn, y y00 =

1Xn=0

(n+ 1) (n+ 2) an+2xn.

Sustituyendo los desarrollos en y0 y y00 en (8), tenemos

1Xn=0

(n+ 1) (n+ 2) an+2xn � (x+ 1)

1Xn=0

anxn = 0

1Xn=0

(n+ 1) (n+ 2) an+2xn �

1Xn=0

anxn+1 �

1Xn=0

anxn = 0

Debemos igualar las potencias de x de la segunda sumatoria, para ello efec-tuamos una operación de asignación a n n� 1, tenemos

1Xn=0

(n+ 1) (n+ 2) an+2xn �

1Xn�1=0

an�1xn�1+1 �

1Xn=0

anxn = 0

1Xn=0

(n+ 1) (n+ 2) an+2xn �

1Xn=1

an�1xn �

1Xn=0

anxn = 0

Desarrollamos un primer término de la primera y tercera sumatoria, respec-tivamente. Entonces, sí n = 0, tenemos

89

1 � 2a2 +1Xn=1

(n+ 1) (n+ 2) an+2xn �

1Xn=1

an�1xn � a0 �

1Xn=1

anxn = 0

Agrupando los términos semejantes, tenemos

(1 � 2a2 � a0) +" 1Xn=1

(n+ 1) (n+ 2) an+2 �1Xn=1

an�1 �1Xn=1

an

#xn = 0

(1 � 2a2 � a0) +1Xn=1

[(n+ 1) (n+ 2) an+2 � an�1 � an]xn = 0

Igualando a cero los coe�entes,se tiene1 � 2a2 � a0 = 0) a2 =

a01 � 2

Luego, la formula de recurrencia es:(n+ 1) (n+ 2) an+2 � an�1 � an = 0

(n+ 1) (n+ 2) an+2 = an�1 + an

an+2 =an�1 + an

(n+ 1) (n+ 2)

Para n = 1; 2; 3; 4; : : :

Tenemos los siguientes coe�cientes

a3 =a0 + a12 � 3 =

a02 � 3 +

a12 � 3

a4 =a1 + a23 � 4 =

a13 � 4 +

a23 � 4 =

a13 � 4 +

a01 � 23 � 4 =

a13 � 4 +

a01 � 2 � 3 � 4

a5 =a2 + a34 � 5 =

a24 � 5 +

a34 � 5 =

a01 � 24 � 5 +

a02 � 3 +

a12 � 3

4 � 5 =a0

1 � 2 � 4 � 5 +a0 + a12 � 3 � 4 � 5

=a0

1 � 2 � 4 � 5 +a0

2 � 3 � 4 � 5 +a1

2 � 3 � 4 � 5 =3a0 + a0

1 � 2 � 3 � 4 � 5 +a15!=4a05!+a15!

...

Sustituyendo estos coe�cientes en al aecuación (9), la solución es:

90

y = a0 + a1x+a02!x2 +

a0 + a13!

x3 +2a1 + a04!

x4 +4a0 + a15!

x5 + � � �

Puntos Singulares.Consideremos la ecuación diferencial líneal de segundo orden

y00 + P (x) y0 +Q (x) y = 0 (1)

El punto x0, es un "punto singular" de la ecuación (1), si una de lasfunciones P (x), Q (x)o ambas a la vez no son analíticas en x0, en caso contrarioes un "punto ordinario" .

Ejemplo.Sea la ecuación

x2y00 + 3xy0 � 2y = 0

Determinamos los punto singulares y ordinarios.Solución.

Primero escribimos la ecuación en la siguiente forma

y00 +3

xy0 � 2

x2y = 0

Donde: P (x) =3

xy Q (x) = � 2

x2, de manera que el punto singular es

x0 = 0. Esto signi�ca que P (x) como Q (x) no son análiticas en el punto x = 0,y los puntos ordinarios son todos los valores de x, excepto x0 = 0:

Ejemplo.Determinar los puntos singulares y ordinarios de la ecuacion de "Bessel"

x2y00 + xy0 +�x2 � n2

�y = 0

donde n es una constante.

Solución.Escribimos la ecuacion de forma

y00 +1

xy0 +

x2 � n2x2

y = 0

y00 +1

xy0 +

�1� n

2

x2

�y = 0

91

Donde P (x) =1

x, Q (x) = 1 � n2

x2. De manera que x = 0 es un punto

singular de la ecuacion de Bessel. Los puntos ordinarios son todos los valoresde x, excepto el cero.

Ejemploconsideramos la ecuación de "Legendre"�

1 + x2�y00 � 2xy0 + a (a+ 1) y = 0

donde a es una constante. Determinar los puntos singulares y regulares.Solución.La ecuación escribimos en su forma más simple.

y00 � 2x

1� x2 y0 +

a (a+ 1)

1� x2 y = 0

Donde: P (x) = � 2x

1� x2 , Q (x) =a (a+ 1)

1� x2 . Para determinar los puntos

singulares de la ecuación de "Legendre", resolvemos la ecuación

1� x2 = 0) x = �1

De manera que los puntos singulares son: x = �1. Los puntos ordinariosson todos los valores de x , excepto �1.

Puntos Singulares Regulares.

De�nición.Se dice que el punto singular x0, de la ecuación (1) , es un "punto singular

regular " si tanto (x� x0)P (x) como (x� x0)2Q (x) son analíticas en x0, encaso contrario se llama punto singular irregular.

Ejemplo.Consideremos la ecuación de "Bessel" en su forma estándar es de la siguiente

manera

y00 +1

xy0 +

x2 � n2x2

y = 0:

92

Clasi�car los puntos singulares.

Solución.El punto singular de la ecuación "Bessel" es x0 = 0. Luego, hacemos la

prueba

(x� x0)P(x) = (x� 0) �1

x= 1

(x� x0)2Q(x) = (x� 0)2�x2 � n2x2

�= x2 � n2:

Esto muestra que x0 = 0 es un punto singular regular.

Ejemplo.Ahora consideremos la ecuación de "Legendre".

y00 � 2x

1� x2 y0 +

a (a+ 1)

1� x2 y = 0:

Clasi�car los puntos singulares.

Solución.

Los punto singulares de la ecuación de Legendre son x0 = �1.

Donde: P (x) = � 2x

1� x2 y Q (x) =a (a+ 1)

1� x2 .

Hacemos la prueba para el punto singular x0 = 1.

(x� x0)P (x) = (x� 1)�� 2x

1� x2

�=�2x (x� 1)� (x2 � 1) =

2x (x� 1)(1 + x) (x� 1) =

2x

1 + x

(x� x0)2Q (x) = (x� 1)2�a (a+ 1)

1� x2

�=a (a+ 1) (x� 1)2

� (1 + x) (x� 1) = �a (a+ 1) (x� 1)

x+ 1

De manera que x0 = 1, es un punto singular regular. En forma análoga severi�ca que el punto singular x0 = �1 es un punto singular regular.

Ejemplo.Clasi�car los puntos singulares de la siguiente ecuación�

x2 � 1�2y00 + (x+ 1) y0 � y = 0

Solución.Primero la ecuación escribimos en forma canónica

93

y00 +(x+ 1)

(x2 � 1)2y0 � 1

(x2 � 1)2y = 0

o simplemente

y00 +1

(x+ 1) (x� 1)2y0 � 1

(x+ 1)2(x� 1)2

y = 0.

Donde P (x) =1

(x+ 1) (x� 1)2y Q(x) = �

1

(x+ 1)2(x� 1)2

Para obtener los puntos singulares, resolvemos la ecuacion.

(x+ 1)2(x� 1)2 = 0:

De manera que los puntos singulares son x0 = �1.

Ahora veri�quemos para x0 = 1

(x� x0)P (x) = (x� 1) �1

(x+ 1) (x� 1)2=

1

(x+ 1) (x� 1)

Para: x0 = 1, resulta

) 1

(x+ 1) (x� 1) =1

0 � 2 =1

0=1

Esto muestra que la función no esta de�nida y que x0 = 1, es un puntosingular irregular.Para x0 = �1.(x� x0)P (x) = (x+ 1) �

1

(x+ 1) (x� 1)2=

1

(x� 1)2

(x� x0)2Q(x) = (x+ 1)2 �"� 1

(x+ 1)2(x� 1)2

#= � 1

(x� 1)2

) � 1

(x� 1)2= � 1

x2 � 2x+ 1 = �1

1 + 2 + 1= �1

4

Esto muestra que x0 = �1; es un punto singular regular.Ejemplo.

Clasi�car los puntos singulares de la ecuación

x3 (1� x) y00 + (3x+ 2) y0 + xy = 0

94

Solución.

Escribimos la ecuación en la forma siguiente

y00 +(3x+ 2)

x3 (1� x)y0 +

x

x3 (1� x)y = 0

y00 +(3x+ 2)

x3 (1� x)y0 +

1

x2 (1� x)y = 0

Como: P (x) =3x+ 2

x3 � x4 y Q (x) =1

x2 (1� x)

Para obtener los puntos singulares , resolvemos la ecuación:

x3 (1� x) = 0

x3 = 0 y 1� x = 0

x = 0 y x = 1

De manera que los puntos singulares son x0 = 0 y x0 = 1.

Veri�camos para x0 = 0

(x� x0)P (x) = (x� 0) �3x+ 2

x3 (1� x) =x (3x+ 2)

x3 (1� x) =3x+ 2

x2 (1 + x)

Para: x0 = 0;resulta3x+ 2

x2 (1 + x)=0 + 2

0=2

0=1

Esto muestra que x0 = 0, es un punto singular irregular.

Veamos para x0 = 1, tenemos

(x� x0)P(x) = (x� 1) �3x+ 2

x3 (1� x) = �3x+ 2

x3

(x� x0)2Q(x) = (x� 1)2 �1

x2 (1� x) = (x� 1)2 1

�x2(x� 1) = �x� 1x2

Como: x0 = 1

�x� 1x2

= �1� 11

= �01= 0

Esto muestra que x0 = 1, es un punto singular regular.

METODO DE FROBENIUSSi x = x0 es un punto singular regular de la ecuación

95

y00 + P (x)y0 +Q(x)y = 0 (1)

entonces, existe al menos una solución en serie de Frobenius de la forma:

y = (x� x0)r1Xn=0

an(x� x0)n =1Xn=0

an(x� x0)n+r

donde r es un número constante, que debe determinarse. Estudiaremossolamente cuando el punto singular regular sea x0 = 0; de manera que

y = xr1Xn=0

anxn =

1Xn=0

anxn+r

La ecuación

r(r � 1) + p0r + q0 = 0,

se llama ecuación indicial de la ecuación (1), cuyas raices son r1 y r2 conr1 > r2 siendo

p0 = limx!x0

(x� x0)P (x) y q0 = limx!x0

(x� x0)2Q(x).

Teorema.La ecuación (1), tiene dos soluciones Linealmente Independientes (L.I.) y1

y y2 si se dan los siguientes casos:CASO I. Si r1 � r2 no es un entero, entonces

y1 =1

xr1X

n=0

anxn; a0 6= 0

y2 = xr2

1Xn=0

bnxn; b0 6= 0.

CASO II. Si r1 = r2, son raices iguales, entonces.

y1 =1

xr1X

n=0

anxn; a0 6= 0

y2 = y1 ln jxj+1

xr2X

n=0

bnxn; b0 6= 0.

96

CASO III. Si r1�r2 = N; donde N es un número entero positivo, entonces

y1 =1Xn=0

anxn+r1 ; a0 6= 0

y2 = Cy1 lnx+1Xn=0

bnxn+r2 ,

donde C es una constante que puede ser cero.

0.1 CASO I.

Ejemplo.Encuentre dos soluciones linealmente independientes usando el método de

Frobenios alrededordel punto x0 = 0, para la ecuación

2xy00 + (1 + x)y0 � 2y = 0.

Solución.La ecuación escribimos en la forma

y00 +1 + x

2xy0 � 1

xy = 0

Donde P (x) =1 + x

2xy Q(x) = � 1

x, vemos que x0 = 0, evidentemente es un

punto singular regular, siendo

p0 = limx!x0

(x� x0)P (x) = limx!0

x

�1 + x

2x

�= lim

x!0

x(1 + x)

2x= lim

x!0

1 + x

2x=1

2,

q0 = limx!x0

(x� x0)2Q(x) = limx!0

x2�� 1x

�= lim

x!0� x

2

x= lim

x!0� x = 0.

De manera que la ecuación inicial es:

r(r � 1) + p0r + q0 = r(r � 1) +1

2r + 0 = r

�r � 1 + 1

2

�= r

�r � 1

2

�= 0.

Igualando cada factor a cero las raices indiciales sonr2 = 0

r1 �1

2= 0) r1 =

1

2.

Como r1 � r2 =1

2� 0 = 1

2, la diferencia de las raices "no es un entero

positivo", entonces las soluciones son de la forma:

97

y = xr1Xn=0

anxn =

1Xn=0

anxn+r

cuyas derivadas son:

y0 =1Xn=0

(n+ r)anxn+r�1

y00 =1Xn=0

(n+ r � 1)(n+ r)anxn+r�2

Sustituyendo y y sus derivadas, en la ecuación inicial, se tiene.

2xy00 + (1 + x)y0 � 2y = 0

2x1Xn=0

(n+r�1)(n+r)anxn+r�2+(1+x)1Xn=0

(n+r)anxn+r�1�2

1Xn=0

anxn+r = 0

21Xn=0

(n+ r � 1)(n+ r)anxn+r�1 +1Xn=0

(n+ r)anxn+r�1 +

1Xn=0

(n+ r)anxn+r �

2

1Xn=0

anxn+r = 0

1Xn=0

(n+ r) [2(n+ r � 1) + 1] anxn+r�1 +1Xn=0

[(n+ r)� 2] anxn+r = 0

1Xn=0

(n+ r)(2n+ 2r � 1)anxn+r�1 +1Xn=0

(n+ r � 2)anxn+r = 0

Ajustamos el índice de la primera sumatoria sustituyendo n � n+ 1,1X

n=�1(n+ r + 1) [2(n+ 1) + 2r � 1] an+1xn+r +

1Xn=0

(n+ r � 2)anxn+r = 0

1Xn=�1

(n+ r + 1)(2n+ 2 + 2r � 1)an+1xn+r +1Xn=0

(n+ r � 2)anxn+r = 0

1Xn=�1

(n+ r+1)(2n+2r+1)an+1xn+r +

1Xn=0

(n+ r� 2)anxn+r = 0.

98

Desarrollamos un término para n = �1 en la primera serie, esto es

r(2r�2+1)a0xr�1+1Xn=0

(n+r+1)(2n+2r+1)an+1xn+r+

1Xn=0

(n+r�2)anxn+r = 0

Agrupamos los coe�cientes de potencias iguales de x,

r(2r� 1)a0xr�1 +1Xn=0

[(n+ r + 1)(2n+ 2r + 1)an+1 + (n+ r � 2)an]xn+r = 0:

(2)

Igualando a cero los coe�cientes de potencias iguales de x, resultar(2r � 1)a0 = 0,

(n+ r + 1)(2n+ 2r + 1)an+1 + (n+ r � 2)an = 0. (3)

Como a0 6= 0 en (2) entonces r(2r�1) = 0 que es la ecuación inicial. Luego

de (3), obtenemos la fórmula de recurrencia

an+1 = �(n+ r � 2)an

(n+ r + 1)(2n+ 2r + 1); n = 0; 1; 2; : : : (4)

Para r = r1 =1

2la ecuación (4) se reduce a

an+1 =

��n+

1

2� 2�an�

n+1

2+ 1

��2n+ 2

�1

2

�+ 1

� = � (2n� 3)an2(n+ 1)(2n+ 3)

; n = 0; 1; 2; : : :

Sí hacemos en esta ecuación n = 0; 1; 2; : : :, nos da

a1 = �(2 � 0� 3)a0

2(0 + 1)(2 � 0 + 3) = ��32 � 3a0 =

3

2 � 3a0,

a2 = �(2 � 1� 3)a1

2(1 + 1)(2 � 1 + 3) = ��1

2 � 2 � 5a1 =1

4 � 5

�3

2 � 3

�a0 =

3

2 � 3 � 4 � 5a0,

a3 = �(2 � 2� 3)a2

2(2 + 1)(2 � 2 + 3) = �4� 3

2 � 3 � (4 + 3)a2 = �1

6 � 7

�3

2 � 3 � 4 � 5

�a0 =

� 3

2 � 3 � 4 � 5 � 6 � 7a0,

99

a4 = �(2 � 3� 3)a3

2(3 + 1)(2 � 3 + 3) = �3

8 � 9

�� 3

2 � 3 � 4 � 5 � 6 � 7

�a0 =

32

2 � 3 � 4 � 5 � 6 � 7 � 8 � 9a0,

...En consecuencia la primera solución linealmente independiente de la ecuación

inicial en serie de Frobenius es:

y1 = xr

1Xn=0

anxn = x

12

�a0 +

3

3!a0x+

3

5!a0x

2 � 3

7!a0x

3 +32

9!a0x

4 + :::::

y1 = a0x12

�1 +

3

3!x+

3

5!x2 � 3

7!x3 +

32

9!x4 + :::::

�.

Para r = r1 = 0; la ecuación (4), se reduce a

an+1 = �(n� 2)an

(n+ 1)(2n+ 1); n = 0; 1; 2; : : :

a1 = �(0� 2)a0

(0 + 1)(2 � 0 + 1) =2

1 � 1a0 = 2a0,

a2 =a12 � 3 =

1

2 � 32a0 =1

3a0,

a3 =a23 � 5 = 0,

a4 =a34 � 7 = 0,

...En consecuencia, la segunda solución linealmente independiente es:

y2 = xr

1Xn=0

anxn = x0

�a0 + 2a0x+

1

3a0x

2 + 0 + : : :

�= a0

�1 + 2x+

x2

3

�:

Por tanto, la solución general es:

y = C1y1(x) + C2y2(x).

y = C1x12

�1 +

3

3!x+

3

5!x2 � 3

7!x3 +

3

9!x4 + � � �

�+ C2

�1 + 2x+

x2

3

�:

100

Finalmente, las dos soluciones L.I. de la ecuación inicial en serie de Frobeniusson:

y1 = x12

�1 +

3

3!x+

3

5!x2 � 3

7!x3 +

3

9!x4 + � � �

�y y2 =

a0

�1 + 2x+

x2

3

�.

CASO IIIa) Caso No LogarítmicoEjemplo.

Hallar dos soluciones liealmente independientes usando el método de Frobe-nius al rededor del puntox0 = 0, para la ecuación

x2y00 � 2xy0 + (x2 + 2)y = 0. (1)SoluciónLa ecuacion escribimos de la forma:

y� � 2xy0 +

x2 + 2

x2y = 0

Donde P (x) = � 2xy Q(x) =

x2 + 2

x2.

Como xP (x) = �2 y x2Q(x) = x2+2 , son analíticas en el origen entoncesel punto x0 = 0 es un punto singular regular, siendo p0 = �2 y q0 = 2: Demanera que la ecuación inicial es:

r(r � 1) + p0r + q0 = r(r � 1) � 2r + 2 = r2 � r � 2r + 2 = r2 � 3r + 2 =(r � 2)(r � 1) = 0.

Las raíces iniciales son: r1 = 2 y r2 = 1. Como: r1 � r2 = 2 � 1 = 1 ladiferencia de las raices es un entero positivo, entonces una solución linealmenteindependiente en serie de Frobenius es:

y = xr1Pn=0

anxn =

1Pn=0

anxn+r (2)

Sus derivadas son:

y0 =1Pn=0

(n+ r)anxn+r�1, y00 =

1Pn=0

(n+ r)(n+ r � 1)anxn+r�2 .

Sustituimos (2) y sus derivadas en (1):

x21Pn=0

(n+r)(n+r�1)anxn+r�2 �2x1Pn=0

(n+r)anxn+r�1+(x2+2)

1Pn=0

anxn+r =

0

101

1Pn=0

(n+r)(n+r�1)anxn+r �21Pn=0

(n+r)anxn+r+

1Pn=0

anxn+r+2+2

1Pn=0

anxn+r =

0

Ajustamos el índice de la tercera sumatoria, reemplazando n! n� 2,1Pn=0

(n+r)(n+r�1)anxn+r �21Pn=0

(n+r)anxn+r+

1Pn=2

an�2xn+r+2

1Pn=0

anxn+r =

0

Desarrollamos los términos para n = 0; 1, tenemos

r(r�1)a0xr+(r+1)ra1xr+1+1Pn=2

(n+r)(n+r�1)anxn+r�2ra0xr�2(r+

1)a1xr+1 � 2

1Pn=2

(n+ r)anxn+r +

1Pn=2

an�2xn+r

+2a0xr + 2a1x

r+1 + 21Pn=2

anxn+r = 0

Agrupamos los coe�cientes de potencias iguales de x,

[r(r � 1) � 2r + 2]a0xr + [(r + 1)r � 2(r + 1) + 2]a1xr+1 +1Pn=2

[(n + r)(n +

r � 1)an � 2(n+ r)an + an�2 + 2an]xn+r = 0

Opeaciones Auxiliares(n+r)(n+r�1)an�2(n+r)an+2an = [(n+r)2�(n+r)�2(n+r)+2]an =

[(n+ r)2 � 3(n+ r) + 2]an= [(n+ r)� 2][(n+ r)� 1]an = [n+ r � 2][n+ r � 1]an

Luego

(r�2)(r�1)a0xr+r(r�1)a1xr+1+1Pn=2

[(n+r�2)(n+r�1)an+an�2]xn+r = 0Igualando a cero los coe�cientes de potencias iguales de x, tendremos(r � 2)(r � 1)a0 = 0r(r � 1)a1 = 0 (3)(n+ r � 2)(n+ r � 1)an + an�2 = 0 para n � 2.

Como: a0 6= 0 (Por Hipótesis), entonces tenemos la ecuación indicial que es:

(r � 2)(r � 1) = 0Ahora pasamos a analizar la ecuación (3) evaluando las raices indiciales en

la ecuación (3), es decir,Para r = r1 = 2, en la ecuacion (3) resulta2(2� 1)a1 = 2 � 1a1 = 2 � a1 = 0, entonces a1 = 0.

Para r = r2 = 1; en la ecuacion (3) resulta1(1 � 1)a1 = 1 � 0a1 = 0 � a1 = 0, entonces a1 6= 0. Esto signi�ca que a1 es

arbitrario.

102

Con a1 6= 0, y n � 2, obtenemos la fórmula de recurencia.

an = �1

(n+ r � 2)(n+ r � 1)an�2 para n � 2. (4)

Para determinar dos soluciones linealmente independientes de la ecuación(1) para el caso r1 = r2 + N , donde N es un entero positivo, se recomiendatomar la raíz menor.Para r = r2 = 1, la ecuación (4), se reduce a

an = �1

(n� 1)nan�2 para n � 2. (5)

Hacemos n = 2; 3; 4; : : : y a1 6= 0 en (5)

a2 = � 11�2a0,

a3 = � 12�3a1,

a4 = � 13�4a2 = �

13�4�� 11�2a0

�= 1

1�2�3�4a0,a5 = � 1

4�5a3 = �14�5�� 12�3a1

�= 1

2�3�4�5a1,...

Por tanto, la solución general es:

y = xr(a0 + a1x+ a2x2 + a3x

3 + : : :)

y = a0x�1� x2

2! +x4

4! � : : :�+ a1x

�x� x3

3! +x5

5! � : : :�.

Esta solución generales es de la forma y = C1y1(x) + C2y2(x).

Finalmente, las dos soluciones L.I. de la ecuación (1) en serie de Frobeniusson:

y1 = x1Pn=0

(�1)n x2n

(2n)!, y2 = x

1Pn=0

(�1)n x2n+1

(2n+ 1)!.

Ejemplo

Hallar dos soluciones liealmente independientes usando el método de Frobe-nius al rededor del puntox0 = 0, para la ecuación

x2(1 + 2x)y00 + 2x(1 + 6x)y0 � 2y = 0 (6)SoluciónLa ecuacion escribimos de la forma:

y00+2(1 + 6x)

x(1 + 2x)y0 � 2

x2(1 + 2x)y = 0.

103

Donde P (x) =2(1 + 6x)

x(1 + 2x)y Q(x) = � 2

x2(1 + 2x).

Como xP (x) =2(1 + 6x)

(1 + 2x)y x2Q(x) = � 2

(1 + 2x), son analíticas en el

origen entonces el punto x0 = 0 es un punto singular regular, siendo p0 = 2 yq0 = �2: De manera que la ecuación inicial es:r(r � 1) + p0r + q0 = r(r � 1) + 2r � 2 = r2 � r + 2r � 2 = r2 + r � 2 =

(r � 1)(r + 2) = 0.

Las raíces iniciales son: r1 = 1 y r2 = �2. Como: r1�r2 = 1�(�2) = 3 ladiferencia de las raices es un entero positivo, entonces una solución linealmenteindependiente en serie de Frobenius es:

y = xr1Pn=0

anxn =

1Pn=0

anxn+r (7)

Sus derivadas son:

y0 =1Pn=0

(n+ r)anxn+r�1, y00 =

1Pn=0

(n+ r)(n+ r � 1)anxn+r�2.

Sustituimos (7) y sus derivadas en (6):

x2(1+2x)1Pn=0

(n+ r)(n+ r�1)anxn+r�2+2x(1+6x)1Pn=0

(n+ r)anxn+r�1�

21Pn=0

anxn+r = 0

1Pn=0

(n+ r)(n+ r� 1)anxn+r + 21Pn=0

(n+ r)(n+ r� 1)anxn+r+1 + 21Pn=0

(n+

r)anxn+r + 12

1Pn=0

(n+ r)anxn+r+1 � 2

1Pn=0

anxn+r = 0

Ajustamos el índice de la segunda y la cuarta sumatoria, reemplazando n!n� 1,

1Pn=0

(n+ r)(n+ r � 1)anxn+r + 21Pn=1

(n+ r � 1)(n+ r � 2)an�1xn+r

+21Pn=0

(n+ r)anxn+r + 12

1Pn=1

(n+ r � 1)an�1xn+r � 21Pn=0

anxn+r = 0

Desarrollamos los primeros términos para n = 0, tenemos

r(r � 1)a0xr +1Pn=1

(n + r)(n + r � 1)anxn+r + 21Pn=1

(n + r � 1)(n + r �

2)an�1xn+r + 2ra0x

r + 21Pn=1

(n+ r)anxn+r

+121Pn=1

(n+ r � 1)an�1xn+r � 2a0xr � 21Pn=1

anxn+r = 0.

Agrupamos los coe�cientes de potencias iguales de x.

[r(r�1)+2r�2]a0xr+1Pn=1

[(n+r)(n+r�1)an+2(n+r�1)(n+r�2)an�1+

104

2(n+ r)an + 12(n+ r � 1)an�1 � 2an]xn+r = 0Opeaciones Auxiliares(n+r)(n+r�1)an+2(n+r)an�2an = [(n+r)2�(n+r)+2(n+r)�2]an =

[(n+ r)2 + (n+ r)� 2]an =(n+ r � 1)(n+ r + 2)an.Por otra parte2(n+r�1)(n+r�2)an�1+12(n+r�1)an�1 = 2(n+r�1)(n+r�2+6)an�1 =

2(n+ r � 1)(n+ r + 4)an�1.Luego:

(r � 1)(r + 2)a0xr +1Pn=1

[(n + r � 1)(n + r + 2)an + 2(n + r � 1)(n + r +

4)an�1]xn+r = 0.

Igualando a cero los coe�cientes de potencias iguales de x, tendremos(r � 1)(r + 2)a0 = 0,(n+ r� 1)(n+ r+ 2)an + 2(n+ r� 1)(n+ r+ 4)an�1 = 0 para n � 1.

(8)

Como a0 6= 0 (por Hipótesis), entonces tenemos la ecuación indicial que es:(r � 1)(r + 2) = 0.

Para determinar dos soluciones linealmente independientes de la ecuación(6), para el caso el caso r1 = r2 + N , donde N es un entero positivo, serecomienda tomar la raíz menor r = r2 = �2, con a0 6= 0 y analizamoscon cuidado el comportamiento de la ecuación de recurrencia (8). Así que, laecuación (8) se reduce a

(n� 3)nan + 2(n� 3)(n+ 2)an�1 = 0, n � 1. (9)

Para n 6= 3 despejamos an y obtenemos la fórmula de recurrencia

an = �2(n+ 2)

nan�1 para n � 1 y n 6= 3. (10)

Hacemos n = 1, 2 en (9)

a1 = �2 � 3a0,a2 = � 2�42 a1 = �4(�2 � 3a0) = 2 � 3 � 4a0.Para n = 3, la ecuación (9) toma la forma 0 � a3 + 0 � a2 = 0, vemos que a3

no puede obtenerse hasta que n > 3. Entonces a3 elegimos como la segundaconstante arbitraria para obtener los demás coe�cientes. Hacemos n = 4; 5; 6; : : :

en (10)a4 = � 2�64 a3 = �3a3,a5 = � 2�75 a4 = �

2�75 (�3a3) =

2�3�75 a3,

a5 = � 2�86 a4 = �83

�2�3�75 a3

�= � 2�7�85 a3,

...Por tanto, la solución general es:

105

y = a0x�2 �1� 6x+ 24x2�+ a3x�2 �x3 � 3x4 + 42

5 x5 � 112

5 x6 + : : :

Finalmente, las dos soluciones L.I. de la ecuación (1) en serie de Frobeniusson:

y1 = x�2 �1� 6x+ 24x2� y y2 = x

�2 �x3 � 3x4 + 425 x

5 � 1125 x

6 + : : :�.

**************** Corregir este ejemplo *****************************************************************************Ejemplo.Determinar dos soluciones Linealmente Independientes, por el método de

Frobenius, alrededor del punto singular regular x0 = 0, de la ecuación

xy00 + 2y0 + 9xy = 0 (1)

Solución.

Escribimos la ecuación de la siguiente forma:

y00 +2

xy0 + 9y = 0

Donde: P (x) =2

xy Q(x) = 9. Vemos que x0 = 0; es un punto singular

regular de la ecuación (1) siendo

p0 = limx!x0

(x� x0)P (x) = limx!0

(x� x0)2

x= lim

x!0

2x

x= lim

x!02 = 2

q0 = limx!x0

(x� x0)2Q(x) = limx!0

(x� x0)29 = limx!0

9x2 = 0

De manera que la ecuación inicial esr(r � 1) + p0r + q0 = 0

r(r � 1) + 2r + 0 = 0

r (r � 1 + 2) = 0

r (r + 1) = 0

r1 = 0

r2 + 1 = 0 r2 = �1

Como r1�r2 = 0�(�1) = 1, la diferencia de las raices es un entero positivo,entonces una solución linealmente independiente es:

106

y = xr1Xn=0

anxn =

1Xn=0

anxn+r

Sus derivadas son:

y0 =1Xn=0

(n+ r)anxn+r�1

y00 =1Xn=0

(n+ r � 1)(n+ r)anxn+r�2.

Sutituyendo y y sus derivadas en (1), tenemos

x1Xn=0

(n+ r � 1)(n+ r)anxn+r�2 + 21Xn=0

(n+ r)anxn+r�1 + 9x

1Xn=0

anxn+r = 0

1Xn=0

(n+ r � 1)(n+ r)anxn+r�1 + 21Xn=0

(n+ r)anxn+r�1 + 9

1Xn=0

anxn+r+1 = 0

1Xn=0

(n+ r) [(n+ r � 1 + 2)] anxn+r�1 + 91Xn=0

anxn+r+1 = 0

1Xn=0

(n+ r)(n+ r + 1)anxn+r�1 + 9

1Xn=0

anxn+r+1 = 0

Haciendo: n n+ 1 y n n� 1, respectivamente, tenemos

1Xn=�1

(n+ r + 1)(n+ r + 2)an+1xn+r + 9

1Xn=1

an�1xn+r = 0

Desarrollamos dos términos para la primeria serie, tenemosPara r = r1 = 0, se tiene

107

r(r + 1)a0xr�1 + (r + 1)(r + 2)a1x

r +1Xn=1

(n+ r + 1)(n+ r + 2)an+1xn+r +

91Xn=1

an�1xn+r = 0

[r(r + 1)a0xr]x�1 + (r + 1)(r + 2)a1x

r +1Xn=1

(n+ r + 1)(n+ r + 2)an+1xn+r +

91Xn=1

an�1xn+r = 0

�r(r + 1)a0x

�1�xr + (r+ 1)(r+ 2)a1xr + 1Xn=1

(n+ r+ 1)(n+ r+ 2)an+1xn+r +

91Xn=1

an�1xn+r = 0

r(r + 1)a0x�1 + [r(r + 1)a0 + (r + 1)(r + 2)a1]x

r +

1Xn=1

[(n+ r + 1)(n+ r + 2)an+1 + 9an�1]xn+r = 0

Igualando a cero los coe�cientes de potencias iguales de x, tendremosr(r + 1)a0 = 0

r(r + 1)a0 + (r + 1)(r + 2)a1 = 0 como r = 0, entonces a1 = 0:

(n+ r + 1)(n+ r + 2)an+1 + 9an�1 = 0

La ecuación inicial es r(r+ 1) = 0, entonces r1 = 0 y r2 = �1. Nótese que

esta ecuación indicial es el mismo que se obtuvo más arriba.

Como se veri�ca las raices son las mismas anteriormente encontradas. Siendo

a1 =�r(r + 1)a0(r + 1)(r + 2)

=�ra0r + 2

Para n � 1; la fórmula de recurrencia general es:

an+1 =�9an�1

(n+ r + 1)(n+ r + 2)=

�32an�1(n+ r + 1)(n+ r + 2)

; n = 1; 2; 3; : : :

Para la raíz más pequeña r2 = �1, tenemos

108

an+1 =�32an�1

(n� 1 + 1)(n� 1 + 2) =�32an�1n(n+ 1)

; n = 1; 2; 3; : : :

Siendo a1 = �ra0r + 2

:

Para r = r2 = �1; tenemos:a1 = �

�1a0�1 + 2 = �

�a01

= a0

Sí: n = 1; 2; 3; : : :

a2 = �32a0

1(1 + 1)= �3

2

2a0 = �

32

2!a0

a3 = �32a1

2(2 + 1)= � 32

2 � 3a0 = �32

3!a0

a4 = �32a2

3(3 + 1)= � 32

3 � 4

��3

2

2!a0

�= � 32

3 � 4

�� 32

1 � 2

�a0 =

34

1 � 2 � 3 � 4a0 =

34

4!a0� � � � � � � � � � � � � � � � � � � � � � � � � � �

Sustituyendo estos coe�cientes en la primera ecuación obtenemos

y1 = xr2

1Xn=0

anxn = x�1

�a0 + a1x+ a2x

2 + a3x3 + � � �

y1 = x�1�a0 + a0x�

32

2!a0x

2 � 32

3!a0x

3 +34

4!a0x

4 + � � ��

y1 =1

xa0

�1 + x� 3

2

2!x2 � 3

2

3!x3 +

34

4!x4 + � � �

y1 = a0

�1

x+ 1� 3

2

2!x� 3

2

3!x2 +

34

4!x3 + � � �

�:

La segunda solución Linealmente Independiente podemos obtener medianteel método de reducción de orden conociendo una solución y1.

y2 = y1

Z1

y21e�RP (x)dxdx

109

Siendo P (x) =2

x; de manera que:

y2 = y1

Z1

y21e�R 2xdxdx = y1

Z1

y21e�2R 1xdxdx = y1

Z1

y21e�2 lnjxjdx

y2 = y1

Z1

y21elnjxj

�2dx = y1

Z1

y21x�2dx = y1

Z1

y21� 1x2dx = y1

Z1

y21x2dx

y2 = y1

Z1

x2

"a0

�1

x+ 1� 3

2

2!x� 3

2

3!x2 +

34

4!x3 + � � �

�2#dx

y2 = y1a0

Z1

x2�1

x+ 1� 9

2x� 3

2x2 +

27

8x3 + � � �

�2 dx

y2 = a0y1

Z1

x2�1

x2+ 1 +

81

4x2 +

9

4x4 +

729

64x6 + � � �+ 2

x� 9� 3x+ 27

4x2 + � � � � 9x� 3x2 + � � �

�dx

y2 = a0y1

Z1

x2�1

x2+2

x� 8� 11x+ 27x2 + � � �

�dx

y2 = a0y1

Z1

1 + 2x� 8x2 � 11x3 + 27x4 + � � �dx

y2 = a0y1

Z �1�1 + 2� 8x� 11x2 + 27x3 + � � �

�dx

y2 = a0y1

�Z1�1dx+ 2

Zdx� 8

Zxdx� 11

Zx2dx+ 27

Zx3 + � � �

�dx

y2 = a0y1

�x+ 2x� 4x2dx� 11

3x3 +

27

4x4 + � � �

�.

TEMA No 5

LA TRANSFORMADA DE LAPLACE.

DEFINICIÓNSea f (t) una función continua en un intervalo [0;1) y suponemos que f

satisface ciertas condiciones. Entonces la integral

110

Lff (t)g = F (s) =Z 1

0

e�stf (t) dt

se denomina "Transformada de Laplace" de f , siempre y cuando la integralsea convergente.

Notación.Sean f (t), g (t) y y (t) funciones continuas, cuyas transformadas de Laplace

son

Lff (t)g = F (s), Lfg (t)g = G (s), Lfy (t)g = y (s)

CONDICIONES SUFICIENTESLas condiciones su�cientes que garantizán la existencia de la transformada de

f (t), Lff (t)g, son que f sea continua parte por parte, en [0;1) y que f sea deorden exponencial para t > T . La primera condición su�ciente, geometricamentesigni�ca que:

ORDEN EXPONENCIAL

De�nición.Se dice que una función f , es de orden exponencial sí existen números a > 0,

M > 0 y T > 0 tales que jf (t)j 6Meat para t > T .Así por ejemplo, sí f es una función creciente, entonces la condición jf (t)j 6

Meat, t > T , simplemente establece que la grá�ca de f en un intervalo [T;1)no crece más rapido que la grá�ca de la función exponencial Meat, donde a esuna constante positiva, gra�camente esto es.

111

OrdenExp

14:pdf

Ejemplo.Las funciones f (t) = t, f (t) = e�t y f (t) = 2 cos t son de orden exponencial

para t > 0, puesto que se veri�ca.

jtj 6 et, je�tj 6 et y j2 cos tj 6 2et

Gra�camente esto signi�ca

Orden Lineal

15:pdf

Orden Exp Neg

16:pdf

Una función tal como f (t) =et2 no es del orden exponencial puesto que sugrá�ca crece más rapido que cualquier función exponencial Meat para a > 0,gra�camente esto es

NoOrdenExp

18:pdf

112

L Es una Transformación LinealTeorema.Supongase que f (t) y g (t) son dos funciones continuas, cuyas transformadas

de Laplace existen, para s > �, s > �, entonces

L [�f (t) + �g (t)] =Z 1

0

e�st [�f (t) + �g (t)] dt =

Z 1

0

e�stf (t) dt+�

Z 1

0

e�stg (t) dt = �Lff (t)g+�Lfg (t)g = �F (s)+�G (s) :

Debido a esta propiedad dada, se dice que L es una transformación lineal,siempre que ambas integrales sean convergentes

Ejemplo 1.

Sea:f (t) = 1 con, t > 0; hallar Lff (t)g por de�nición:

Solución.

Lf1g =Z 1

0

e�st (1) dt = limb!1

Z b

0

e�stdt = limb!1

e�st

�s jb0 = lim

b!1

��e

�bs

s+1

s

�=

� limb!1

e�bs

s+ limb!1

1

s= 0 +

1

s.

De manera que F (s) =1

s, s > 0

Ejemplo 2.

Sea f (t) = t con, t > 0,hallar Lff (t)g por de�nición.Solución.

Donde Lff (t)g =Z 1

0

e�sttdt

Integrando por partes, tenemos

Sí: u = t) du = dt

dv = e�st ) v =

Ze�stdt = �e

�st

s

Lff (t)g =Z 1

0

e�sttdt = �te�st

sj10 +

Z 1

0

e�st

sdt = �te

�st

sj10 +

1

s

Z 1

0

e�stdt =

�0 + 0� 1s

e�st

sj10 , Sí: e�st = 0, cuando t!1, entonces

113

: =1

s

��0 + 1

s

�F (s) =

1

s2, s > 0 .

Ejemplo 3.

Sea: f (t) = t2 con , t > 0;hallar Lff (t)g por de�nición:Solución.

Donde L�t2=

Z 1

0

e�stt2dt

Integrando por partes

Sí: u = t2 ) du = 2t dt

dv = e�st ) v =

Ze�stdt = �e

�st

s

L�t2=

Z 1

0

e�stt2dt = �t2 e�st

sj10 +

2

s

Ze�sttdt = �0 + 0 + 2

sLftg =

2

s

�1

s2

�=2

s3; s > 0.

En general

Lftng = n!

sn+1, para todo n = 1; 2; 3; : : :

Ejemplo 4.

Sea: f (t) = e�2t con , t > 0;hallar Lff (t)g por de�nición:Solución.

Donde L�e�2t

=

Z 1

0

e�ste�2tdt =

Z 1

0

e�(s+2)tdt = �e�(s+2)t

(s+ 2)j10 = �0+

1

s+ 2.

F (s) =1

s+ 2, s > �2.

Ejemplo 5.

Sea: f (t) = sen at con, t > 0: Hallar Lff (t)g por de�nición:

Solución.

Donde Lfsen atg =Z 1

0

e�st sen at dt

114

Integrando por partes, tenemos

Sí: u = e�st ) du = �se�stdtdv = sen at) v =

Zsen at dt = �cos at

a.

Luego,

Lfsen atg =Z 1

0

e�st sen at dt = �e�st cos ataj10 �

s

a

Z 1

0

e�st cos at dt .

Integrando nuevamente por partes, tenemos

Sí: u = e�st ) du = �se�stdtdv = cos at) v =

Zcos atdt =

sen at

a.

De donde

Lfsen atg =Z 1

0

e�st sen at dt = �0+1a� sa

�e�st

sen at

aj10 +

s

a

Z 1

0

e�st sen at dt

�=1

a� sa

�0� 0 + s

a

Z 1

0

e�st sen at dt

�Lfsen atg = 1

a� s

2

a2Lfsen atg

L fsen atg+ s2

a2Lfsen atg = 1

a�1 +

s2

a2

�Lfsen atg = 1

a

Lfsen atg = 1

a

�a2 + s2

a2

� .

De manera queF (s) =

a

s2 + a2, s > a .

Ejemplos.Utilizando la tabla y la linealidad de la transformada de Laplace, determinar

la transformada de la siguientes funciones.

1). Sea: f (t) = 3t3 � 2t2 + t� 5Lff (t)g = L

�3t3 � 2t2 + t� 5

= 3L

�t3� 2L

�t2+ Lftg � 5Lf1g =

115

3

�3!

s4

�� 2

�2!

s3

�+ 1

�1

s2

�� 5

�1

s

�F (s) =

18

s4� 4

s3+1

s2� 5s, s > 0 .

2). Sea: f (t) = (t� 1)3

Lff (t)g = Ln(t� 1)3

o= L

�t3 � 3t2 + 3t� 1

= L

�t3� 3L

�t2+

3Lftg � Lf1g = 3!

s4� 3

�2!

s3

�+ 3

�1

s2

�� 1s

F (s) =6

s4� 6

s3+3

s2� 1s, s > 0 .

3). Sea: f (t) = 4e�5t + 3 sen t� 2 cos 4tLff (t)g = L

�4e�5t + 3 sen t� 2 cos 4t

= 4L

�e�5t

+3Lfsen tg�2Lfcos 4tg

= 4

�1

s� 5

�+ 3

�1

s2 + 12

�� 2

�s

s2 + 42

F (s) =4

s� 5 +3

s2 + 12� 2s

s2 + 42, s > �5 .

4). Sea: f (t) = e�2t�t4 + 2t2 � 1

�Lff (t)g = L

�e�2t

�t4 + 2t2 � 1

�= L

�e�2tt4 + 2e�2tt2 � e�2t

= L

�t4e�2t

+2L

�t2e�2t

�L

�e�2t

=

4!

(s+ 2)5 +2

"2!

(s+ 2)3

#�

1

s+ 2

F (s) =24

(s+ 2)5 +

4

(s+ 2)3 �

1

s+ 2, s > �2 .

5). Sea: f (t) = e2t sen 3t

Lff (t)g = L�e2t sen 3t

=

3

(s� 2)2 + 32=

3

s2 � 4s+ 4 + 9

F (s) =3

s2 � 4s+ 13 .

6). Sea: f (t) = e�6t cosp2t� 1

2t sen 2

p3t+ 5

116

Lff (t)g = L�e�6t cos

p2t� 1

2t sen 2

p3t+ 5

= L�e�6t cos

p2t� 12L�t sen 2

p3t+ 5Lf1g

=s+ 6

(s+ 6)2+�p2�2 � 12

0B@ 4p3sh

s2 +�2p3�2i2

1CA+ 5�1s

F (s) =s+ 6

s2 + 12s+ 38� 2

p3s

(s2 + 12)2 +

5

s, s > 0 .

7). Sea: f (t) = sen 4t cos 4t

Lff (t)g = Lfsen 4t cos 4tg = 1

2Lf2 sen 4t cos 4tg = 1

2Lfsen 8tg = 1

2

�8

s2 + 82

F (s) =4

s2 + 64.

8). Sea: f (t) = cos 2t cos tLff (t)g = Lfcos 2t cos tg = L

��cos2 t� sen2 t

�cos t

= L

�cos3 t� sen2 t cos t

= L

�cos3 t

�L

�sen2 t cos t

= L

�cos 3t+ 3 cos t

4

��L

��1� cos 2t

2

�cos t

�=1

4Lfcos 3tg+ 3

4Lfcos tg � 1

2Lfcos tg+ 1

2Lfcos 2t cos tg

Luego, escribimos

Lfcos 2t cos tg � 12Lfcos 2t cos tg =

1

4Lfcos 3tg+ 3

4Lfcos tg � 1

2Lfcos tg

12Lfcos 2t cos tg =

1

4Lfcos 3tg+ 3

4Lfcos tg � 1

2Lfcos tg

=1

4Lfcos 3tg+ 3

4Lfcos tg � 1

2Lfcos tg

=1

4

�s

s2 + 32

�+3

4

�s

s2 + 12

�� 12

�s

s2 + 12

�=

s

4 (s2 + 9)+1

4

�s

s2 + 12

�.

Por tanto,Lfcos 2t cos tg = s

2 (s2 + 9)+

s

2 (s2 + 1).

TRANSFORMADA INVERSA DE LAPLACE

117

Ahora estudiamos el proceso inverso de la sección anterior , es decir dadauna función F (s) hallar la función f(t), que corresponde a su transformada. Sedice que f(t) es la transformada inversa de F (s), entonces escribimos:

f(t) = L�1fF (s)g ,

L�1es una Transformación Lineal.

La transformada inversa de laplace es en si misma una transformación lineal,esto es, para las constantes � y �, entonces:

L�1 f�F (s) + �G (s)g = �L�1 fF (s)g+ �L�1 fG (s)g

donde, F y G son las transformadas de algunas funciones f y g .

Ejemplos.

Dada las siguientes funciones, determinar su transformada inversa.

1). Sea: F (s) =4

s2 + 9

L�1fF (s)g = L�1�

4

s2 + 9

�=4

3L�1

�3

s2 + 32

�.

Entonces:f(t) =

4

3sen 3t

2). Sea: F (s) =4s

4s2 + 1

L�1fF (s)g = L�1�

4s

4s2 + 1

�= L�1

8><>:4s

44s2 + 1

4

9>=>; = L�1(

s

s2 +�12

�2).

Entonces:

f(t) = cos1

2t .

3). Sea: F (s) =2s� 6s2 + 9

L�1fF (s)g = L�1�2s� 6s2 + 9

�= 2L�1

�s

s2 + 32

�� 2L�1

�3

s2 + 32

�.

118

Entonces:

f(t) = 2 cos 3t� 2 sen 3t = 2(cos 3t� sen 3t) .

4). Sea F (s) =s+ 1

s2 + 2s+ 10

L�1fF (s)g = L�1�

s+ 1

s2 + 2s+ 10

�= L�1

�s+ 1

(s2 + 2s+ 12) + 10� 12

�=

L�1(

s+ 1

(s+ 1)2+ 32

).

Entonces:f(t) =e�t cos 3t

5). Sea: F (s) =1

s2 + s� 20

L�1fF (s)g = L�1�

1

s2 + s� 20

�= L�1

8>><>>:1�

s2 + s+1

4

���20 +

1

4

�9>>=>>; =

2

9L�1

8>>><>>>:9

2�s+

1

2

�2��9

2

�29>>>=>>>; .

Entonces:f(t) =

2

9e�

12 t senh 9

2 t .

6). Sea: F (s) =3s� 15

2s2 � 4s+ 10L�1fF (s)g = L�1

�3s� 15

2s2 � 4s+ 10

Factorizando tenemos:

L�1fF (s)g = 3

2L�1

�s� 5

s2 � 2s+ 5

�=3

2L�1

�s� 5

(s2 � 2s+ 12) + 5� 12

�=

3

2L�1

�(s� 1)� 4(s� 1)2 + 22

�=3

2L�1

�(s� 1)� 4(s� 1)2 + 22

�=

3

2L�1

�(s� 1)

(s� 1)2 + 22

�� 322L�1

�2

(s� 1)2 + 22

�=3

2L�1

�(s� 1)

(s� 1)2 + 22

�� 3L�1

�2

(s� 1)2 + 22

�.

119

Entonces:f(t) =

3

2et cos 2t� 3et sen 2t .

7). Sea: G(S) =2s+ 16

s2 + 4s+ 13

L�1fG(s)g = L�1�

2s+ 16

s2 + 4s+ 13

�= L�1

�2s+ 16

(s2 + 4s+ 22) + 13� 22

= 2L�1�

s+ 8

(s2 + 4s+ 22) + 9

�= 2L�1

�(s+ 2) + 6

(s+ 2)2 + 32

= 2L�1�

s+ 2

(s+ 2)2 + 32

�+ 4L�1

�3

(s+ 2)2 + 32

�= 2L�1

�s+ 2

(s+ 2)2 + 32

�+ 4L�1

�3

(s+ 2)2 + 32

�.

Entonces:g(t) = 2

�e�2t cos 3t+ 2e�2t sen 3t

�.

8). Sea: G(s) =8s2

s4 + 8s2 + 16

L�1fG(s)g = L�1�

8s2

s4 + 8s2 + 16

�= L�1

�8s2

(s2 + 22)2

�= L�1

�2 � 2 � 2s2(s2 + 22)2

�= 2L�1

�(2 � 2s2)(s2 + 22)2

�. Por

L�1fsen kt+ kt cos ktg = 2ks2

(s2 + k2)2.

Entonces:

g(t) = 2 (sen 2t+ 2t cos 2t) .

FRACCIONES PARCIALES

Los fracciones parciales, son muy importantes para determinar la transfor-mada inversa de Laplace. Así consideramos los ejemplos siguientes

EjemplosDeterminar la transformada inversa de la función:

F (s) =s2 � 26s� 47

(s� 1)(s+ 2)(s+ 5) .

120

Solución.

Aplicamos fracciones parciales a la función f(s).

s2 � 26s� 47(s� 1)(s+ 2)(s+ 5) =

A

s� 1 +B

s+ 2+

C

s+ 5

=A(s+ 2)(s+ 5) +B(s� 1)(s+ 5) + C(s� 1)(s+ 2)

(s� 1)(s+ 2)(s+ 5)s2 � 26s� 47 = A(s+ 2)(s+ 5) +B(s� 1)(s+ 5) + C(s� 1)(s+ 2)

Evaluando

Sí: s = 1, tenemos(1)2 � 26(1)� 47 = A(3)(6)

18A = �72

A = � 7218

A = �4

Sí:s = �2, tenemos(�2)2 � 26(�2)� 47 = B(�3)(3)

�9B = 9

B = �1 .

Sí: s = �5, tenemos(�5)2 � 26(�5)� 47 = C(�6)(�3)

18C = 108

C = 6

Siendos2 � 26s� 47

(s� 1)(s+ 2)(s+ 5) =�4s� 1 +

�1s+ 2

+6

s+ 5:

Luego

L�1�

s2 � 26s� 47(s� 1)(s+ 2)(s+ 5)

�= L�1

��4s� 1

�+L�1

��1s+ 2

�+L�1

�6

s+ 5

121

= �4L�1�

1

s� 1

��L�1

�1

s+ 2

�+6L�1

�1

s+ 5

�Entonces

f(t) = �4et � e�2t + 6e�5t.

Ejemplo

Determinar L�1 fF (s)g, donde

F (s) =s2 + 8

s4 � 4s2

Solución.

Primero factorizamos el denominador de F (s)

F (s) =s2 + 8

s4 � 4s2

Aplicando fracciones parciales a F (s) , tenemos:s2 + 8

s2(s+ 2)(s� 2) =A

s+B

s2+

C

s+ 2+

D

s� 2

=As(s+ 2)(s� 2) +B(s+ 2)(s� 2) + Cs2(s� 2) +Ds2(s+ 2)

s2(s+ 2)(s� 2)s2 + 8 = As(s+ 2)(s� 2) +B(s+ 2)(s� 2) + Cs2(s� 2) +Ds2(s+ 2)

Hacemos la evaluación

Sí: s = 0, tenemos8 = B(2)(�2)

B = �2

Sí: s = �2, entonces(�2)2 + 8 = C(�2)2(�4)

16C = �12

C = �34

Sí: s = 2, tenemos

122

22 + 8 = D(4)(4)

16D = 12

D =3

4

Ahora efectuamos la siguiente operacións2 + 8 = As3 � 4As�Bs2 � 4B � Cs3 � 2Cs2 +Ds3 + 2Ds2

= (A+B +D)s3 + (B � C + 2D)s2 � 4As� 4B .

Igualando los coe�cientes de potencias iguales de s, obtenemos el siguientesistema8>><>>:

A+ C +D = 0B � 2C + 2D = 1�4A = 0�4B = 8

=> A = 0 B = �2:

Siendo entoncess2 + 8

s2(s+ 2)(s� 2) =0

s+�2s2+� 34s+ 2

+34

s� 2

Luego

L�1�

s2 + 8

s2(s+ 2)(s� 2)

�= �2 L�1

�1

s2

��34L�1

�1

s+ 2

�+3

4L�1

�1

s� 2

�Entonces

f(t) = �2t� 34%�2t + 3

4%2t .

Ejemplo

Determinar L�1 fF (s)g, donde

F (s) =2s3 + s2 + 3

s4 � 1

Solución.

Factorizar el denominador de F (s):

F (s) = L�1�

2s3 + s2 + 3

(s+ 1)(s� 1)(s2 + 1)

Aplicando fracciones parciales a F (s), se tiene

123

2s3 + s2 + 3

(s+ 1)(s� 1)(s2 + 1) =A

(s+ 1)+

B

s� 1 +Cs+D

s2 + 1

=A(s� 1)(s2 + 1) +B(s+ 1)(s2 + 1) + (s+ 1)(s� 1)(Cs+D)

(s+ 1)(s� 1)(s2 + 1)2s3 + s2 + 3 = A(s� 1)(s2 + 1) +B(s+ 1)(s2 + 1) + (s+ 1)(s� 1)(Cs+D)

2s3+s2+3 = As3+As�As2�A+Bs3+Bs+Bs2+B+Cs3�Cs+Ds2�D

= (A+B+C)s3+(�A+B+D)s2+(A+B�C)s+(�A+B�D)

.

Igualando los coe�cientes de potencias iguales de s ,Obtenemos el sistema8>><>>:A+B + C = 2 (1)�A+B +D = 1 (2)A+B � C = 0 (3)�A+B �D = 3 (4)

Resolviendo el sistema obtenemos que: A = � 12 ; B =32 ; C = 1; D = �1:

Luego2s3 + s2 + 3

(s+ 1)(s� 1)(s2 + 1) =� 12s+ 1

+32

s� 1 +s� 1s2 + 1

:

Siendo

L�1�

2s3 + s2 + 3

(s+ 1)(s� 1)(s2 + 1)

�= �1

2L�1

�1

s+ 1

�+3

2L�1

�1

s� 1

�+L�1

�s

s2 + 1

��

L�1�

1

s2 + 1

�:

Entoncesf(t) = � 12e

�t + 32et + cos t� sen t .

LA TRANSFORMADA DE LAPLACE DE LA DERIVADA

Teorema A.Sea f (x) una función continua en [0;1) y que f 0 (x) es continua parte por

parte en [0;1) siendo ambos de orden exponencial a. Entonces, para s > a

Lff 0 (t)g = sLff (t)g � f (0)

Demostración.Para demostrar este Teorema, suponemos que existe Lff 0 (t)g, siendo

124

Lff 0 (t)g =Z 1

0

e�stf 0 (t) dt.

Integrando por partes, tenemos

Sí: u =e�st ) du = �se�stdtdv = f 0 (t)) v =

Zf 0 (t) dt = f (t).

Prosiguiendo

Lff 0 (t)g =Z 1

0

e�stf 0 (t) dt =e�stf (t) j10 + s

Z 1

0

e�stf (t) dt = �f (0) +

sLff (t)g.

En consecuencia

Lff 0 (t)g = sLff (t)g � f (0).

ó bien

Lff 0 (t)g = sF (s)� f (0).

donde

F (s) = Lff (t)g :

Nótese que, e�stf (t) = 0 cuando t!1.En forma análoga podemos obtener Lff 00 (t)g. Vemos que la transformada

es de la siguiente manera

Lff 00 (t)g =Z 1

0

e�stf 00 (t) dt.

125

Integrando por partes

Sí: u =e�st ) du = �se�stdtdv = f 00 (t) dt) v = f 0 (t).

Prosiguiendo

Lff 00 (t)g =Z 1

0

e�stf 00 (t) dt =e�stf 0 (t) j10 +

Z 1

0

e�stf 0 (t) dt = �f

0 (0) + s [sLff (t)g � f (0)]

En consecuencia

Lff 00 (t)g = s2Lff (t)g � sf (0)� f 0 (0).

En general, resulta

L�f (n) (t)

= snLff (t)g � sn�1f (0)� sn�2f 0 (0)� � � � � f (n�1) (0)

donde

F (s) = Lff (t)g :

Ejemplo.

Determinar Lfcos atg, utilizando el Teorema A..Solución.

Sea: f (t) = cos at

Donde f (0) = cos a0 = 1 y f 0 (t) = �a sen at, luego sustituimos en elTeorema A.

Lf�a sen atg = sLfcos atg � 1

�aLfsen atg = sLfcos atg � 1

�a�

a

s2 + a2

�= sLfcos atg � 1

sLfcos atg = 1� a2

s2 + a2

Lfcos atg = 1

s� a2

s (s2 + a2)=s2 + a2 � a2s (s2 + a2)

Lfcos atg = s2

s (s2 + a2)=

s

s2 + a2.

126

APLICACIONESResolución de Problemas de Valor InicialLa transformada de Laplace, se puede aplicar para resolver problemas de

valor inicial a ecuaciones diferenciales lineales con coe�cientes constantes.

Ejemplo.Utilizando la transformada de Laplace, resolver el problema de valor inicial

y0 + 3y =e�3t, y (0) = 4

Solución.Aplicando la transformada de Laplace a ambos miembros de la ecuación,

tenemos

Lfy0 + 3yg = L�e�3t

Aplicando linealidad de la transformada, se tiene

Lfy0g+ 3Lfyg = L�e�3t

sLfyg � y (0) + 3Lfyg = L

�e�3t

sY (s)� 4 + 3Y (s) = 1

s+ 3

Y (s) (s+ 3)� 4 = 1

s+ 3

Y (s) (s+ 3) =1

s+ 3+ 4

Y (s) =1

(s+ 3)2 +

4

s+ 3.

Ahora, calculamos la transformada inversa de Y (s), y esto es

L�1 fY (s)g = L�1(

1

(s+ 3)2 +

4

s+ 3

)

L�1 fY (s)g = L�1(

1

(s+ 3)2

)+ 4L�1

�1

s+ 3

�.

Por tanto, la solución es

y (t) = te�3t + 4e�3t.

127

Ejemplo.Utilizando la transformada de Laplace , resolver el problema de valor inicial.

y00 � 4y0 + 4y = t3e2t ; y (0) = 0 , y0 (0) = 0

Solución.

Aplicamos L y linealidad

Lfy00g � 4Lfy0g+ 4Lfyg = L�t3e2t

s2Lfyg � sy (0)� y0 (0)� 4 (sLfyg � y (0)) + 4Lfyg = L

�t3e2t

s2Y (s)� 4sY (s) + 4Y (s) = 6

(s� 2)4

Y (s)�s2 � 4s+ 4

�=

6

(s� 2)4

Y (s) (s� 2)2 = 6

(s� 2)4

Y (s) =6

(s� 2)6.

Ahora, calculamos L�1 y la linealidad,

L�1 fY (s)g = L�1(

6

(s� 2)6

)

L�1 fY (s)g = 6L�1(

1

(s� 2)6

)

L�1 fY (s)g = 6

5!L�1

(5!

(s� 2)6

)

y (t) =1

20t5e2t.

Ejemplo.

Utilizando la transformada de Laplace, resolver el problema de valor inicial

128

y00 + 2y0 + 5y = 3e�t sen t; y (0) = 0 , y0 (0) = 3.

Solución.

Aplicando L y linealidad, tenemos

Lfy00g+ 2Lfy0g+ 5Lfyg = 3Lfe�t sen tg

s2Lfyg � sy (0)� y0 (0) + 2 (sLfyg � y (0)) + 5Lfyg = 3Lfe�t sen tg

s2Y (s)� 3 + 2sY (s) + 5Y (s) = 3"

1

(s+ 1)2+ 12

#

s2Y (s) + 2sY (s) + 5Y (s) =3

s2 + 2s+ 2+ 3

Y (s)�s2 + 2s+ 5

�=

3

s2 + 2s+ 2+ 3

Y (s) =3

(s2 + 2s+ 5) (s2 + 2s+ 2)+

3

s2 + 2s+ 5.

Aplicando fracciones parciales, tenemos3

(s2 + 2s+ 5) (s2 + 2s+ 2)=

As+B

s2 + 2s+ 5+

Cs+D

s2 + 2s+ 2=(As+B)

�s2 + 2s+ 2

�+ (Cs+D)

�s2 + 2s+ 5

�(s2 + 2s+ 5) (s2 + 2s+ 2)

=As3 + 2As2 + 2As+Bs2 + 2Bs+ 2B + Cs3 + 2Cs2 + 5Cs+Ds2 + 2Ds+ 5D

(s2 + 2s+ 5) (s2 + 2s+ 2)

=(A+ C) s3 + (2A+B + 2C +D) s2 + (2A+ 2B + 5C + 2D) s+ (2B + 5D)

(s2 + 2s+ 5) (s2 + 2s+ 2):

3

(s2 + 2s+ 5) (s2 + 2s+ 2)=(A+ C) s3 + (2A+B + 2C +D) s2 + (2A+ 2B + 5C + 2D) s+ (2B + 5D)

(s2 + 2s+ 5) (s2 + 2s+ 2).

Igualando los numeradores, se tiene0s3+0s2+0s+3 = (A+ C) s3+(2A+B + 2C +D) s2+(2A+ 2B + 5C + 2D) s+

(2B + 5D).

Igualando, los coe�cientes de potencias iguales de s del primer y segundomiembro, obtenemos el sistema8>><>>:

A+ C = 0 (1)2A+B + 2C +D = 0 (2)2A+ 2B + 5C + 2D = 0 (3)2B + 5D = 3 (4)

De la primera ecuación despejamos C = �A y reemplasando en la segunadaecuación del sistema, obtenemos

129

0.1.1 2A+B � 2A+D = 0) B +D = 0.

Luego, resolvemos el sistemaB +D = 0 j �52B + 5D = 3 j� � � � � � ��5B � 5D = 02B + 5D = 3� � � � � � ��3B = 3 ) B = �1. Demanera que: D = 1:

Luego, reemplazando B = �1 y D = 1 en la tercera ecuación del sitema, seobtiene2A+ 5C = 0.Sustituyendo C = �A, en esta ecuación, se obtiene2A+ 5 (�A) = 0 ) A = 0 y C = 0.

En consecuencia, las raices del sistema son: A = 0, B = �1, C = 0 yD = 1.Siendo

3

(s2 + 2s+ 5) (s2 + 2s+ 2)=

�1s2 + 2s+ 5

+1

s2 + 2s+ 2ContinuandoY (s) =

�1s2 + 2s+ 5

+1

s2 + 2s+ 2+

3

s2 + 2s+ 5=

2

s2 + 2s+ 5+

1

s2 + 2s+ 2

De manera que

L�1 fY (s)g = L�1�� 1

s2 + 2s+ 5

�+L�1

�1

s2 + 2s+ 2

�+L�1

�3

s2 + 2s+ 5

= L�1�

2

s2 + 2s+ 5

�+ L�1

�1

s2 + 2s+ 2

= L�1�

2

s2 + 2s+ 5

�+ L�1

�1

s2 + 2s+ 2

= L�1�

2

(s2 + 2s+ 12) + 5� 12

�+L�1

�1

(s2 + 2s+ 12) + 2� 12

= L�1(

2

(s+ 1)2+ 4

)+ L�1

(1

(s+ 1)2+ 1

)

130

L�1 fY (s)g = L�1(

2

(s+ 1)2+ 4

)+ L�1

(1

(s+ 1)2+ 1

).

Finalmente:y (t) = e�t sen 2t+ e�t sen t .

Ejemplo.Mediante la transformada de Laplace, resolver el problema de valor inicial.

y00 � y0 � 2y = �2 sen t� 8 cos t ; y��2

�= 1 , y0

��2

�= 0.

Solución.Donde

Lfy00g � Lfy0g � 2Lfyg = �2Lfsen tg � 8Lfcos tg

s2Lfyg � sy��2

�� y0

��2

��hsLfyg � y

��2

�i� 2Lfyg =

�2Lfsen tg � 8Lfcos tg

s2Y (s)� s� [sY (s)� 1]� 2Y (s) = �2�

1

s2 + 12

�� 8

�s

s2 + 12

�s2Y (s)� s� sY (s) + 1� 2Y (s) = �2

�1

s2 + 12

�� 8

�s

s2 + 12

��s2 � s� 2

�Y (s)� s+ 1 = �2

s2 + 12+�8ss2 + 12

(s� 2) (s+ 1)Y (s) = �2� 8ss2 + 1

+ s� 1

Y (s) =�2� 8s

(s� 2) (s+ 1) (s2 + 1) +s� 1

(s� 2) (s+ 1)

Y (s) =�8s� 2 + (s� 1)

�s2 + 1

�(s� 2) (s+ 1) (s2 + 1) =

�8s� 2 + s3 + s� s2 � 1(s� 2) (s+ 1) (s2 + 1)

Y (s) =s3 � s2 � 7s� 3

(s� 2) (s+ 1) (s2 + 1) .

Aplicando fraccciones parciales, tenemoss3 � s2 � 7s� 3

(s� 2) (s+ 1) (s2 + 1) =A

s� 2 +B

s+ 1+Cs+D

s2 + 1

=A (s+ 1)

�s2 + 1

�+B (s� 2)

�s2 + 1

�+ (Cs+D) (s� 2) (s+ 1)

(s� 2) (s+ 1) (s2 + 1)

=(As+A)

�s2 + 1

�+ (Bs� 2B)

�s2 + 1

�+�Cs2 +D

� �s2 � s� 2

�(s� 2) (s+ 1) (s2 + 1)

131

=As3 +As+As2 +A+Bs3 +Bs� 2Bs2 � 2B + Cs3 � Cs2 � 2Cs+Ds2 �Ds�D

(s� 2) (s+ 1) (s2 + 1)

=As3 +Bs3 + Cs3 +As2 � 2Bs2 � Cs2 +Ds2 +As+Bs� 2Cs�Ds+A� 2B � 2D

(s� 2) (s+ 1) (s2 + 1)

=(A+B + C) s3 + (A� 2B � C +D) s2 + (A+B � 2C �D) s+ (A� 2B � 2D)

(s� 2) (s+ 1) (s2 + 1) .

o biens3 � s2 � 7s� 3

(s� 2) (s+ 1) (s2 + 1) =(A+B + C) s3 + (A� 2B � C +D) s2 + (A+B � 2C �D) s+ (A� 2B � 2D)

(s� 2) (s+ 1) (s2 + 1) .

Simpli�cando los denominadores, se tienes3�s2�7s�3 = (A+B + C) s3+(A� 2B � C +D) s2+(A+B � 2C �D) s+

(A� 2B � 2D) (1)

Evaluación:

Sí: s = 2 en (1), entonces23�22�7 (2)�3 = (A+B + C) 23+(A� 2B � C +D) 22+(A+B � 2C �D) 2+

(A� 2B � 2D)8 � 4 � 14 � 3 = 8A + 8B + 8C + 4A � 8B � 4C + 4D + 2A + 2B � 4C �

2D +A� 2B � 2D�13 = 15A)) A = �13

15.

Sí: s = �1 en (1), entonces�1�1+7�3 = (A+B + C) (�1)+(A� 2B � C +D)+(A+B � 2C �D) (�1)+

(A� 2B � 2D)2 = �A�B � C +A� 2B � C +D �A�B + 2C +D +A� 2B � 2D

2 = �6B )) B = �13.

Luego, igualando los coe�cientes de potencias iguales de s en (1), obtenemosel sistema8>><>>:

A+B + C = 1 (1)A� 2B � C +D = �1 (2)A+B � 2C �D = �7 (3)A� 2B � 2D = �3 (4)

132

Como ya conocemos de antemano las raíces A = �1315

y B = �13, susti-

tuyendo estos valores en el sistema obtenemos los demás raices, esto es: C =33

15

y D =7

5.

Operaciones Auxiliares:Sea la ecuaciónA+B + C = 1

�1315� 13+ C = 1

C = 1 +13

15+1

3)) C = 33

15.

Reemplazamos en la segunda ecuación los valores de A, B ;C y D, tenemosA� 2B � C +D = �1�1315� 2

��13

�� 3315+D = �1

�1315+2

3� 3315+D = �1

�13 + 10� 3315

+D = �110� 4615

+D = �1

�3615+D = 1

D =36

15� 1

D =36� 1515

=21

15)) D =

7

5.

Continuamos, reemplazando los valores de A, B, C y D en la función Y (s):

Y (s) =s3 � s2 � 7s� 3

(s� 2) (s+ 1) (s2 + 1) =�1315

s� 2 +�13

s+ 1+

33

15s+

7

5s2 + 1

Y (s) = �1315

�1

s� 2

�� 13

�1

s+ 1

�+33

15

�s

s2 + 1

�+7

5

�1

s2 + 1

�.

Ahora calculamos la transformada inversa de Y (s)

L�1 fY (s)g =

�1315L�1

�1

s� 2

�� 13L�1

�1

s+ 1

�+33

15L�1

�s

s2 + 1

�+7

5L�1

�1

s2 + 1

�.

En consecuencia, la solución es:

y (t) = �1315e2t� 1

3e�t+33

15cos t+

133

7

5sen t .

Ejemplo.

Mediante la transformada de Laplace, resolver el problema de valor incial.

y00 + y0 � 2y = cos 2t ; y (0) = 1 , y0 (0) = 4.

Solución.Lfy00g+ Lfy0g � 2Lfyg = Lfcos 2tg

s2Lfyg � sy (0)� y0 (0) + sLfyg � y (0)� 2Lfyg = L�cos2 t

�L

�sen2 t

s2Y (s)� s� 4 + sY (s)� 1� 2Y (s) = s2 + 2

s (s2 + 4)� 2

s (s2 + 4)

�s2 + s� 2

�Y (s)� s� 5 = s2 + 2

s (s2 + 4)� 2

s (s2 + 4)

(s+ 2) (s� 1)Y (s) = s2

s (s2 + 4)+ s+ 5

Y (s) =s2

s (s2 + 4) (s+ 2) (s� 1) +s+ 5

(s+ 2) (s� 1)

Simpli�cando s en la primera fracción y despejando Y (s), se tiene

Y (s) =s

(s2 + 4) (s+ 2) (s� 1) +s+ 5

(s+ 2) (s� 1)

Y (s) =s+

�s2 + 4

�(s+ 5)

(s2 + 4) (s+ 2) (s� 1) =s+ s3 + 5s2 + 4s+ 20

(s2 + 4) (s+ 2) (s� 1) .Simpli�cando los términos semejantes del numerador, resulta

Y (s) =s3 + 5s2 + 5s+ 20

(s2 + 4) (s+ 2) (s� 1) .

Aplicando fracciones parciales, tenemoss3 + 5s2 + 5s+ 20

(s2 + 4) (s+ 2) (s� 1) =As+B

s2 + 4+

C

s+ 2+

D

s� 1 .

Operamos en el segundo miembro

As+B

s2 + 4+

C

s+ 2+

D

s� 1 =(As+B) (s+ 2) (s� 1) + C

�s2 + 4

�(s� 1) +D

�s2 + 4

�(s+ 2)

(s2 + 4) (s+ 2) (s� 1)

=(As+B)

�s2 + s� 2

�+ C

�s3 � s2 + 4s� 4

�+D

�s3 + 2s2 + 4s+ 8

�(s2 + 4) (s+ 2) (s� 1)

134

=As3 +As2 � 2As+Bs2 +Bs� 2B + Cs3 � Cs2 + 4Cs� 4C +Ds3 + 2Ds2 + 4Ds+ 8D

(s2 + 4) (s+ 2) (s� 1)

=As3 + Cs3 +Ds3 +As2 +Bs2 � Cs2 + 2Ds2 � 2As+Bs+ 4Cs+ 4Ds� 2B � 4C + 8D

(s2 + 4) (s+ 2) (s� 1)

=(A+ C +D)s3 + (A+B � C + 2D)s2 + (�2A+B + 4C + 4D)s+ (�2B � 4C + 8D)

(s2 + 4) (s+ 2) (s� 1) .

O biens3 + 5s2 + 5s+ 20

(s2 + 4) (s+ 2) (s� 1) =(A+ C +D)s3 + (A+B � C + 2D)s2 + (�2A+B + 4C + 4D)s+ (�2B � 4C + 8D)

(s2 + 4) (s+ 2) (s� 1)

Simpli�camos los denominadores

s3 + 5s2 + 5s+ 20 = (A+ C +D)s3 + (A+ B � C + 2D)s2 + (�2A+ B +4C + 4D)s+ (�2B � 4C + 8D)Igualando los coe�cientes de potencias iguales s del primer y segundo miem-

bro, tenemos el siguiente sistema8>><>>:A+ C +D = 1 (1)A+B � C + 2D = 5 (2)�2A+B + 4C + 4D = 5 (3)�2B � 4C + 8D = 20 (4)

Resolviendo el sistema encontramos que la raíces son: A =3

20, B =

1

10,

C = �1112, D =

31

15.

Continuamos, reemplazando los valores de A, B, C y D en la función Y (s):

Y (s) =320s+

110

s2 + 4�

1112

s+ 2+

3115

s� 1

Y (s) =3

20

�1

s2 + 4

�+1

10

�1

s2 + 4

�� 1112

�1

s+ 2

�+31

15

�1

s� 1

�.

Ahora calculamos la transformada inversa de Y (s)

L�1 fy (s)g = 3

20L�1

�s

s2 + 4

�+1

10L�1

�1

s2 + 4

�� 11

12L�1

�1

s+ 2

�+

31

15L�1

�1

s� 1

�.

En consecuencia, la solución es:

135

y (t) =3

20cos 2t+ 1

20 sin 2t�11

12e�2t + 31

15et.

136